Q1: 'A democracy is more than a form of government, it is a mode of associated living', who said this?
(a) Jawaharlal Nehru
(b) John Locke
(c) John Dewey
(d) Gandhi
Ans: c
Sol: The correct answer is option 3, John Dewey.
- John Dewey was an American philosopher, psychologist, and educational reformer who believed that democracy was not just a political system, but a way of life. He argued that democracy required active participation from citizens and emphasized the importance of education in creating informed and engaged citizens.
Other Related Points
- Option 1, Jawaharlal Nehru, was the first Prime Minister of India and a prominent leader in the Indian independence movement. He believed in democratic socialism and worked to establish a democratic government in India.
- Option 2, John Locke, was an English philosopher who is known for his ideas on natural rights and the social contract. He believed that government should be based on the consent of the governed and that individuals had the right to life, liberty, and property.
- Option 4, Gandhi, was an Indian independence activist who believed in nonviolent resistance and civil disobedience. He worked to establish a democratic government in India and believed in the importance of individual freedom and self-rule.
Overall, all of these figures believed in the importance of democracy and individual freedom, but John Dewey's quote emphasizes the idea that democracy is not just a political system, but a way of life that requires active participation and engagement from citizens.
Q2: Sophie or Sophia is a character in which of the following books?
(a) Republic
(b) A Vindication of the rights of women
(c) The Archaeology of knowledge
(d) The human condition
Ans: b
Sol: The correct answer is option 2, "A Vindication of the Rights of Women" by Mary Wollstonecraft.
- Sophie is not a character in any of the other options.
- "A Vindication of the Rights of Women" is a feminist text published in 1792 that argues for women's education and equal rights. Sophie is used as an example of how women are often raised to be decorative objects rather than educated individuals. Wollstonecraft argues that women should be given the same opportunities as men to develop their minds and contribute to society.
Other Related Points
- Option 1, "Republic" by Plato, is a philosophical text that discusses the nature of justice and the ideal society. There is no character named Sophie in this book.
- Option 3, "The Archaeology of Knowledge" by Michel Foucault, is a philosophical text that explores the history and development of knowledge. There is no character named Sophie in this book.
- Option 4, "The Human Condition" by Hannah Arendt, is a philosophical text that examines the nature of human existence and the meaning of life. There is no character named Sophie in this book.
Q3: Who among the following has used the terms internal restriction and external restriction in the discourse of multiculturalism?
(a) Bhikhu Parekh
(b) Will Kymlicka
(c) Okin
(d) Brian Barry
Ans: b
Sol: The correct answer is option 2, Will Kymlicka.
- Kymlicka is a Canadian political philosopher who has extensively written on multiculturalism.
- He has used the terms internal restriction and external restriction to describe the challenges faced by minority cultures in a multicultural society.
- Internal restrictions refer to the limitations imposed by the minority culture on its own members, while external restrictions refer to the limitations imposed by the dominant culture on the minority culture.
Other Related Points
- Option 1, Bhikhu Parekh, is also a political philosopher who has written on multiculturalism. He has emphasized the importance of dialogue and mutual respect between different cultures.
- Option 3, Okin, is Susan Moller Okin, a feminist philosopher who has written on multiculturalism and gender equality. She has criticized multiculturalism for sometimes allowing for the oppression of women within minority cultures.
- Option 4, B. Berry, is a sociologist who has written on multiculturalism and the challenges of managing diversity in society. He has emphasized the importance of recognizing and valuing cultural differences while also promoting social cohesion.
Q4: For whom among the following M.B. Foster has stated that his political thought represents more of a Pagan revival than a Protestant religion?
(a) Machiavelli
(b) Hobbes
(c) Bentham
(d) J.S. Mill
Ans: a
Sol: The correct answer is option 1, Machiavelli.
- M.B. Foster, a political theorist, has argued that Machiavelli's political thought represents more of a Pagan revival than a Protestant religion.
- Foster believes that Machiavelli's emphasis on the importance of power and the state's ability to use force to maintain order is more in line with ancient pagan beliefs than with Protestant ideas of morality and individual rights.
Other Related Points
- Option 2, Hobbes, is known for his social contract theory and belief in the absolute power of the state to maintain order. However, his political thought is not typically associated with Paganism.
- Option 3, Bentham, is known for his utilitarianism, which emphasizes the greatest good for the greatest number of people. His political thought is not typically associated with Paganism.
- Option 4, J.S. Mill, is known for his ideas about individual liberty and the importance of limiting the power of the state. His political thought is not typically associated with Paganism.
Overall, Foster's argument about Machiavelli's political thought representing a Pagan revival is a unique interpretation that challenges traditional views of Machiavelli's ideas.
Q5: Which one of the following is not Aristotle's work?
(a) Politics
(b) Nicomachean
(c) Eudemian Ethics
(d) Apology
Ans: d
Sol: The correct answer is option 4, Apology.
- Apology is not a work of Aristotle, but rather a work of Plato.
- It is a dialogue that depicts the trial of Socrates and his defense against the charges of impiety and corrupting the youth.
Other Related Points
- Option 1, Politics, is a work of Aristotle that discusses the nature of the state, different forms of government, and the ideal state.
- Option 2, Nicomachean Ethics, is another work of Aristotle that explores the nature of ethics and the good life.
- Option 3, Eudemian Ethics, is also a work of Aristotle that discusses ethics and the virtues, but it is less well-known than Nicomachean Ethics.
Q6: The 'Analects' is related to :
(a) Nicomachean
(b) Plato
(c) Confucius
(d) Chou dynasty
Ans: c
Sol: The correct answer is option 3, Confucius.
- Confucius was a Chinese philosopher and teacher who lived during the Spring and Autumn period of Chinese history.
- He emphasized the importance of moral values, social order, and respect for authority. His teachings, known as Confucianism, have had a profound influence on Chinese culture and society.
Other Related Points
- Option 1, Nicomachean, was a Greek mathematician and philosopher who lived in the 2nd century AD. He is known for his work on arithmetic and music theory.
- Option 2, Plato, was a Greek philosopher who lived in the 4th century BC. He was a student of Socrates and the teacher of Aristotle. He is known for his philosophical works, including The Republic and The Symposium.
- Option 4, the Chou dynasty, was a Chinese dynasty that ruled from the 11th century BC to the 3rd century BC. It was a period of great cultural and intellectual development in China, and is known for its contributions to literature, philosophy, and art.
Q7: "Our souls have been corrupted in proportion to the advancement of our sciences and our arts towards perfection". It is mentioned in:
(a) Discourse on the Science and Arts
(b) Revolt against reason
(c) The Wealth of Nations
(d) The Theory of Moral Sentiments
Ans: a
Sol: The correct answer is option 1, "Discourse on the Science and Arts".
- This is a work by philosopher Jean-Jacques Rousseau, in which he argues that the arts and sciences have corrupted human morality and society.
- He suggests that a return to a simpler, more natural way of life is necessary for true happiness and virtue.
Other Related Points
- Option 2, "Revolt against reason," is not a specific work or concept, so it is not a correct answer.
- Option 3, "The Wealth of Nations," is a work by economist Adam Smith, in which he argues for the benefits of free market capitalism and division of labor.
- Option 4, "The Theory of Moral Sentiments," is another work by Adam Smith, in which he explores the nature of morality and the role of sympathy in human behavior.
Q8: Which one of the following is perfect, eternal and fixed according to Plato?
(a) Matter
(b) Ideas
(c) Raw Material
(d) Darkness
Ans: b
Sol: The correct answer is option 2, Ideas.
- According to Plato, Ideas or Forms are perfect, eternal and fixed. They exist in a realm beyond the physical world and are the true reality.
- Matter, on the other hand, is constantly changing and imperfect. Raw material is simply the material that is used to create something and is not perfect or eternal.
- Darkness is a lack of light and has no qualities of perfection or eternity. Plato believed that the physical world is a mere shadow or imitation of the true reality of the world of Forms.
- The Forms are perfect and unchanging, while the physical world is constantly changing and imperfect.
- Plato believed that knowledge of the Forms is necessary for true understanding and wisdom.
Q9: When and at which place in South Africa the Indians assembled, demonstrated and took the oath of passive resistance?
(a) September 1906, Johannesburg
(b) October 1906, Natal
(c) September 1905, Pretoria
(d) October 1905, Pietermaritzburg
Ans: a
Sol: The correct answer is option 1, September 1906 in Johannesburg.
- This event is known as the Satyagraha Campaign, where Indians in South Africa gathered to protest against the discriminatory laws imposed on them by the government.
- They took an oath of passive resistance, pledging to disobey unjust laws and accept the consequences of their actions.
- This event marked the beginning of a long struggle for Indian rights in South Africa.
Other Related Points
- Option 2, October 1906 in Natal, is incorrect as there is no record of any significant Indian demonstration or oath-taking event in Natal during that time.
- Option 3, September 1905 in Pretoria, is also incorrect as there is no record of any major Indian protest or demonstration in Pretoria during that time.
- Option 4, October 1905 in Pietermaritzburg, is incorrect as there was a significant event that took place in Pietermaritzburg, but it was not related to the Satyagraha Campaign. In June 1893, Mahatma Gandhi was thrown off a train at Pietermaritzburg station for refusing to move from a first-class compartment reserved for whites. This incident inspired Gandhi to fight against racial discrimination in South Africa, but it was not an oath-taking event or a demonstration.
Q10: Who among the following launched the 'Self-Respect Movement' in 1925?
(a) Dr. B.R. Ambedkar
(b) Mahatma Phule
(c) E.V. Ramaswami Naicker
(d) J.P. Narayan
Ans: c
Sol: The correct answer is option 3, E.V. Ramaswami Naicker.
- He launched the Self-Respect Movement in 1925 in Tamil Nadu, India. The movement aimed to uplift the socially and economically oppressed sections of society, particularly the Dalits and non-Brahmins.
- Naicker advocated for the eradication of caste-based discrimination and the promotion of rationalism and self-respect among these communities.
Other Related Points
- Option 1, Dr. B.R. Ambedkar, was a prominent Dalit leader and social reformer who fought against caste-based discrimination and worked towards the upliftment of the Dalit community. He was also the architect of the Indian Constitution.
- Option 2, Mahatma Phule, was a social reformer and activist who worked towards the education and empowerment of women and the lower castes in Maharashtra, India.
- Option 4, J.P. Narayan, was a political leader and activist who played a key role in the Indian independence movement and later in the fight against the authoritarian regime of Indira Gandhi in the 1970s.
Overall, all of these leaders played important roles in the social and political movements of India, working towards greater equality and justice for all.
Q11: Who among the following categorized laws into two types: the Shariat and the Zawabit?
(a) Dara Shukoh
(b) Basava
(c) Abul Fazl
(d) Ziauddin Barani
Ans: d
Sol: Option 4, Ziauddin Barani, is the correct answer.
- Zia Barani was a 14th-century Persian historian who wrote the book "Tarikh-i-Firuz Shahi," which is a historical account of the reign of Sultan Firuz Shah Tughlaq.
- The book provides valuable insights into the social, economic, and political conditions of medieval India.
Other Related Points
- Option 1, Dara Shukoh, was a Mughal prince and the eldest son of Emperor Shah Jahan. He was known for his interest in Sufism and comparative religion.
- Option 2, Basava, was a 12th-century Indian philosopher and social reformer who founded the Lingayat sect. He advocated for social equality and rejected the caste system.
- Option 3, Abul Fazl, was a courtier and historian in the court of Emperor Akbar. He is known for his work "Akbarnama," which is a detailed account of Akbar's reign.
Q12: When did M.N. Roy dismantle his Radical Democratic Party ?
(a) 1940
(b) 1945
(c) 1948
(d) 1950
Ans: c
Sol: The correct answer is option 3, 1948.
- M.N. Roy dismantled his Radical Democratic Party in 1948 due to internal conflicts and disagreements among party members.
- Roy had founded the party in 1940 with the aim of promoting socialist and democratic ideals in India.
- However, the party failed to gain significant support and faced criticism for its lack of clear political strategy.
Other Related Points
- Option 1, 1940, is incorrect as this was the year when M.N. Roy founded the Radical Democratic Party.
- Option 2, 1945, is also incorrect as there is no significant event or information available to suggest that Roy dismantled his party in this year.
- Option 4, 1950, is incorrect as Roy had already disbanded his party in 1948.
Overall, the dismantling of the Radical Democratic Party marked a significant moment in the political career of M.N. Roy, who continued to be involved in socialist and communist movements in India and abroad.
Q13: Why Robert Nozick agrees with anarchist assertions regarding the position of State? Pick the incorrect argument
(a) It is intrinsically immoral.
(b) It maintains monopoly on the use of force.
(c) It violates individual rights.
(d) Its motive is welfare of citizens.
Ans: d
Sol: The correct answer is option 4, which states that the motive of the State is the welfare of citizens.
- Robert Nozick disagrees with this assertion because he believes that the State's primary function is to protect individual rights, not to promote the welfare of citizens.
- He argues that the State's use of force to enforce its laws and regulations is inherently coercive and violates individual rights.
- Therefore, he agrees with anarchist assertions that the State is intrinsically immoral, maintains a monopoly on the use of force, and violates individual rights.
Other Related Points
- Option 1 is correct because Nozick believes that the State is intrinsically immoral due to its use of force to enforce its laws and regulations.
- Option 2 is correct because Nozick believes that the State maintains a monopoly on the use of force, which is inherently coercive and violates individual rights.
- Option 3 is correct because Nozick believes that the State's use of force to enforce its laws and regulations violates individual rights.
Overall, Nozick's argument is that the State's primary function should be to protect individual rights, not to promote the welfare of citizens. Therefore, he agrees with anarchist assertions that the State is inherently immoral, maintains a monopoly on the use of force, and violates individual rights.
Q14: Which of the following amendment is related to 'anti-defection"?
(a) 51st Amendment
(b) 52nd Amendment
(c) 53rd Amendment
(d) 59th Amendment
Ans: b
Sol: The correct answer is option 2, which is the 52nd Amendment.
- This amendment was passed in 1985 and added the Tenth Schedule to the Indian Constitution, which introduced the concept of "anti-defection".
- This means that if an elected member of a political party defects to another party, they will lose their seat in the legislature.
- The amendment was introduced to prevent political instability caused by frequent defections.
Other Related Points
- Option 1, the 51st Amendment, was passed in 1984 and extended the reservation of seats for Scheduled Castes and Scheduled Tribes in the Lok Sabha and state legislative assemblies.
- Option 3, the 53rd Amendment, was passed in 1986 and added a new article to the Constitution, which gave the state the power to make special provisions for the advancement of any socially and educationally backward classes of citizens.
- Option 4, the 61st Amendment, was passed in 1988 and lowered the voting age from 21 to 18 years.
Q15: Which one of the following is not a principle among the originally framed six principles of 'Citizen's Charter Movement'?
(a) Choice
(b) Value
(c) Accountability
(d) Quantity
Ans: d
Sol: The correct answer is option 4, Quantity.
- The originally framed six principles of the Citizen Charter Movement are Choice, Value, Accountability, Transparency, Accessibility, and Timeliness.
- These principles aim to promote good governance and ensure that public services are delivered efficiently and effectively to citizens.
Other Related Points
- Option 1, Choice, refers to the provision of multiple options for citizens to access public services.
- Option 2, Value, emphasizes the importance of providing high-quality services that meet the needs and expectations of citizens.
- Option 3, Accountability, requires public officials to take responsibility for their actions and be answerable to citizens.
Q16: The four Ps in the 4 Ps theory of departmentalization as advocated by Luther Gulick are ____________.
(a) Purpose, Persons, Place, Process
(b) Persons, Place, Process and Programmes
(c) Process, People, Policies and Perception
(d) People, Public, Problems and Perception
Ans: a
Sol: The correct answer is option 1, which includes Purpose, Persons, Place, and Process. These four Ps are used to categorize and organize different departments within an organization based on their functions and responsibilities.
- Purpose refers to the overall goal or objective of the department, while Persons refer to the individuals who work within the department.
- Place refers to the physical location or area of responsibility of the department, and Process refers to the methods and procedures used to achieve the department's purpose.
Other Related Points
- Option 2 includes Persons, Place, Process, and Programmes, which is similar to option 1 but includes the additional category of Programmes, which refers to the specific projects or initiatives that the department is responsible for.
- Option 3 includes Process, People, Policies, and Perception, which focuses more on the internal workings of the department, including the methods and procedures used, the people involved, the policies in place, and how the department is perceived by others.
- Option 4 includes People, Public, Problems, and Perception, which focuses more on the external factors that impact the department, including the people it serves, the problems it addresses, and how it is perceived by the public.
Q17: According to Follett the difference between 'power' and 'authority' is ________.
(a) Power exists when orders are obeyed irrespective of resistance, authority exists when orders obeyed voluntarily.
(b) Power can never be delegated, authority can be delegated.
(c) Authority exists when orders are obeyed irrespective of resistance, power exists when orders obeyed by force.
(d) Authority can never be delegated, power can always be delegated.
Ans: a
Sol: This question has been dropped by UGC NET.
- Power is the ability to make someone do something even if they don't want to, while authority is the ability to make someone do something because they believe it is the right thing to do.
- Power is often associated with coercion and force, while authority is associated with legitimacy and respect.
Q18: Which is the fifth stage of Policy Cycle according to Harold Lasswell?
(a) Promotion
(b) Invocation
(c) Application
(d) Intelligence
Ans: c
Sol: The correct answer is option 3, which is Application.
- This stage involves the implementation of policies that have been formulated and adopted.
- It involves putting the policies into action and ensuring that they are carried out effectively.
- This stage also involves monitoring and evaluating the policies to determine their effectiveness and making necessary adjustments.
Other Related Points
- Option 1, Promotion, refers to the stage where policies are proposed and advocated for. This involves raising awareness and building support for the policies.
- Option 2, Invocation, refers to the stage where policies are officially adopted and put into effect. This involves the legal and administrative processes of enacting the policies.
- Option 4, Intelligence, refers to the stage where information is gathered and analyzed to inform policy formulation. This involves research, data collection, and analysis to identify problems and potential solutions.
Q19: The literature on New Public Administration lays emphasis on ___________.
(a) Relevance, Relatedness, Moral Values and Equity
(b) Relevance, Values, Ethics and Change
(c) Values, Ethics, Relatedness and Self Reliance
(d) Relevance, Values, Equity and Change
Ans: d
Sol: The correct answer is option 4, which emphasizes the importance of relevance, values, equity, and change in the field of New Public Administration.
- Relevance refers to the need for public administration to address current and pressing issues, while values and ethics guide decision-making and actions.
- Equity ensures fairness and justice in the distribution of resources and services, and change recognizes the need for adaptation and improvement in response to evolving circumstances.
Other Related Points
- Option 1 emphasizes moral values and equity, which are important considerations in public administration, but does not include relevance or relatedness.
- Option 2 includes relevance and values, but does not mention equity or change.
- Option 3 includes values and relatedness, but does not mention relevance or equity.
Q20: Which of the following Ministry is associated with Swachh Survekshan?
(a) Ministry of Jal Shakti
(b) Ministry of Housing and Urban Affairs
(c) Ministry of Rural Development
(d) Ministry of Finance
Ans: b
Sol: The correct answer is option 2, Ministry of Housing and Urban Affairs.
- Swachh Survekshan is an annual survey conducted by the Ministry of Housing and Urban Affairs to assess the cleanliness and sanitation of cities and towns in India.
- The survey evaluates various parameters such as waste management, open defecation, and citizen feedback to rank the cities and towns based on their cleanliness.
Other Related Points
- Option 1, Ministry of Water and Sanitation, is associated with the Swachh Bharat Abhiyan, a national campaign launched to promote cleanliness and sanitation in India.
- Option 3, Ministry of Rural Development, is associated with the Swachh Gramin Abhiyan, a campaign launched to promote cleanliness and sanitation in rural areas of India.
- Option 4, Ministry of Finance, is not associated with Swachh Survekshan or any other cleanliness and sanitation campaign.
Q21: Which of the following is the first state to bring Social Audit Law?
(a) Andhra Pradesh
(b) Meghalaya
(c) Tripura
(d) Chhattisgarh
Ans: b
Sol: The correct answer is 'Meghalaya'.
Meghalaya became the first state in India to bring Social Audit Law in 2017.
- The Meghalaya Community Participation and Public Services Social Audit Act, 2017, aims to ensure transparency and accountability in the implementation of government schemes and programs.
- The law mandates the conduct of social audits by citizens and civil society organizations to assess the effectiveness of government programs and services.
Q22: Which of the following Ministry/Aayog is associated with Good Governance Index?
(a) Ministry of Personnel Public Grievances and Pensions
(b) Ministry of Rural Development
(c) Ministry of Housing and Urban Affairs
(d) NITI Aayog
Ans: a
Sol: The correct answer is option 1, i.e., Ministry of Personnel Public Grievances and Pensions.
- The Good Governance Index (GGI) was launched by the Ministry of Personnel Public Grievances and Pensions in 2019.
- The index aims to assess the state of governance in the country and promote the culture of good governance.
- It evaluates the performance of states and union territories on various parameters such as effective governance, service delivery, economic governance, and citizen-centric governance.
Other Related Points
- Option 2, i.e., Ministry of Rural Development, is associated with various schemes and programs related to rural development such as Mahatma Gandhi National Rural Employment Guarantee Act (MGNREGA), Pradhan Mantri Awaas Yojana (PMAY), etc.
- Option 3, i.e., Ministry of Housing and Urban Affairs, is responsible for the formulation and implementation of policies related to urban development, housing, and urban poverty alleviation.
- Option 4, i.e., NITI Aayog, is a policy think tank of the Government of India that provides strategic and technical advice to the central and state governments on various issues related to economic and social development. It is not directly associated with the Good Governance Index.
Q23: Which of the following Act is also known as 'Hyde Act'?
(a) U.S. - India Economic Co-operation Act
(b) U.S. - India Peaceful Atomic Energy Co-operation Act
(c) U.S. - India Trade Co-operation Act
(d) U.S. - India Arms Trade Act
Ans: b
Sol: The correct answer is option 2, which is the U.S. - India Peaceful Atomic Energy Co-operation Act.
- This act is also known as the Hyde Act, named after its sponsor, Congressman Henry Hyde.
- The act outlines the terms and conditions for the United States to provide nuclear technology and fuel to India for peaceful purposes, while also ensuring that India adheres to non-proliferation standards.
Other Related Points
- Option 1, the U.S. - India Economic Co-operation Act, is not known as the Hyde Act and focuses on promoting economic ties between the two countries.
- Option 3, the U.S. - India Trade Co-operation Act, is also not known as the Hyde Act and aims to enhance trade relations between the United States and India.
- Option 4, the U.S. - India Arms Trade Act, is not a real act and was likely included as a distractor. Overall, the Hyde Act is a significant piece of legislation in the U.S.-India relationship, as it paved the way for greater cooperation in the nuclear energy sector.
Q24: Which are the six minority religious communities notified by the National Commission for Minorities (NCM) setup by the Union Government under the National Commission for Minorities Act 1992?
(a) Muslims, Christians, Sikhs, Buddhists, Zoroastrians, Sindhis
(b) Muslims, Christians, Sikhs, Buddhists, Sindhis, Bahais
(c) Muslims, Christians, Sikhs, Sindhis, Bahais, Jews
(d) Muslims, Christians, Sikhs, Buddhists, Zoroastriansand Jains
Ans: d
Sol: The correct answer is option 4: Muslims, Christians, Sikhs, Buddhists, Zoroastrians and Jains.
- These six religious communities have been notified by the National Commission for Minorities (NCM) under the National Commission for Minorities Act 1992.
- The NCM was set up by the Union Government to safeguard the constitutional and legal rights of minority communities in India.
Other Related Points
- Option 1 includes Sindhis, which is not a minority religious community.
- Option 2 includes Bahais, which is not a recognized minority community in India.
- Option 3 includes Jews, which is not a significant minority community in India. It is important to note that while these six communities are recognized as minority communities by the NCM, there are other religious and linguistic minorities in India that may not be included in this list. The NCM also works to protect the rights of these other minority communities.
Q25: Who among the following conducts the elections at Polling Station?
(a) Polling Agent
(b) Presiding Officer
(c) Returning Officer
(d) Revenue Officer
Ans: b
Sol: The correct answer is option 2, Presiding Officer.
- The Presiding Officer is responsible for conducting the elections at the Polling Station.
- They are appointed by the Election Commission and are in charge of ensuring that the election is conducted in a fair and transparent manner.
- They are responsible for setting up the polling station, verifying the identity of voters, maintaining order and discipline at the polling station, and ensuring that the voting process is conducted smoothly.
Other Related Points
- Option 1, Polling Agent, is a representative of a political party or candidate who is present at the polling station to observe the voting process and ensure that it is conducted fairly.
- Option 3, Returning Officer, is responsible for overseeing the entire election process in a constituency, including the counting of votes and declaring the results.
- Option 4, Revenue Officer, has no role in conducting elections at a polling station. They are responsible for revenue collection and administration in their respective areas.
Q26: Which of the following statements is not correct regarding Union Public Service Commission (UPSC)?
(a) The President of India can remove the Chairman of UPSC.
(b) In case of removal of Chairman, the President has to refer the matter to the Supreme Court for an enquiry.
(c) An Individual Ministry or Department has power to reject the advice of the UPSC.
(d) The State Governments can ask the UPSC to conduct the examinations for concerned state.
Ans: c
Sol: The correct answer is An Individual Ministry or Department has power to the advice of the UPSC..
The Union Public Service Commission (UPSC) is an autonomous body under the Government of India that is responsible for conducting the Civil Services Examination and other competitive examinations for the All India Services and Central Services of the Government of India.
- The UPSC is not under the control of any individual Ministry or Department of the Government of India.
- Therefore, the State Governments can ask the UPSC to conduct the examinations for the concerned state but only with prior permission.
Other Related Points
The other statements are correct.
- The President of India can remove the Chairman of the UPSC after an enquiry by the Supreme Court.
- An Individual Ministry or Department has no power to reject the advice of the UPSC.
- The UPSC is an independent body and its advice is binding on all the Ministries and Departments of the Government of India.
Q27: Which one of the following fundamental duty is inserted by 86th Constitutional Amendment in Article 51-A?
(a) To strive towards excellence in all spheres of individual and collective activity.
(b) To provide opportunities for education by the parents to their child.
(c) To develop the scientific temper, humanism and spirit of inquiry and reform.
(d) Respect to the National Flag and Symbols.
Ans: b
Sol: The correct answer is To provide opportunities for education by the parents to their child.
The 86th Constitutional Amendment Act, 2002 inserted this fundamental duty in Article 51-A of the Constitution of India.
- This amendment was made to provide for the Right to Education for all children between the age of 6 and 14 years.
- The amendment also made it the duty of parents and guardians to provide opportunities for education to their children.
The detailed explanation of the correct answer is as follows:
- The 86th Constitutional Amendment Act, 2002 was passed by the Parliament of India on 20 December 2002.
- The amendment inserted Article 21A in the Constitution of India, which provides for the Right to Free and Compulsory Education for all children between the age of 6 and 14 years.
- The amendment also amended Article 51-A of the Constitution of India, and inserted a new fundamental duty, which is to provide opportunities for education by the parents to their child.
- The amendment was made to ensure that all children in India have access to quality education, regardless of their social or economic background.
- The amendment has been a major step towards achieving the goal of universal education in India.
Q28: Who among the following is pioneer of Public Interest Litigation?
(a) Justice S.N. Banerjee
(b) Justice P.N. Bhagwati
(c) Justice Hingorani
(d) Justice Ranjan Gogoi
Ans: b
Sol: The correct answer is option 2, Justice P.N. Bhagwati.
- He is considered the pioneer of Public Interest Litigation (PIL) in India.
- He introduced the concept of PIL in the 1980s, which allowed any citizen to approach the court on behalf of the public interest.
- This was a significant shift from the traditional approach of litigation, which was limited to parties directly affected by a dispute.
- PIL has since become an important tool for promoting social justice and holding the government accountable.
Other Related Points
- Option 1, Justice S.N. Benarji, is not associated with PIL.
- Option 3, Justice Hingorani, is known for his work on environmental issues and was involved in several PIL cases related to environmental protection.
- Option 4, Justice Ranjan Gogoi, is a former Chief Justice of India who is known for his judgments on issues such as citizenship and the independence of the judiciary. However, he is not particularly associated with PIL.
Q29: Who among the following contributed to the framing of National Rural Employment Guarantee Act?
(a) Jean Drieze
(b) Aruna Roy
(c) Medha Patkar
(d) Baba Amte
Ans: a
Sol: The correct answer is option 1, Jean Drieze.
- He is a Belgian economist who played a crucial role in the conceptualization and design of the National Rural Employment Guarantee Act (NREGA) in India.
- He worked closely with social activists like Aruna Roy and Nikhil Dey to develop the framework of the program, which guarantees 100 days of employment to rural households in India.
Other Related Points
- Option 2, Aruna Roy, is also a social activist who played a key role in advocating for the NREGA and ensuring its implementation. She was part of the National Campaign for People's Right to Information and the Mazdoor Kisan Shakti Sangathan, which worked towards empowering rural communities and ensuring their rights.
- Option 3, Medha Patkar, is an environmental and social activist who is known for her work on issues related to displacement and rehabilitation of communities affected by large-scale development projects. While she may not have directly contributed to the framing of NREGA, her work has been instrumental in highlighting the need for social safety nets for marginalized communities.
- Option 4, Baba Amte, was a social activist and philanthropist who worked towards the rehabilitation of leprosy patients and other marginalized communities. While he may not have been directly involved in the framing of NREGA, his work on social justice and empowerment has inspired many activists and policymakers in India.
Q30: Which of the following statements is correct regarding the vote value of a member of Parliament in presidential election?
(a) 
(b) 
(c) 
(d) 
Ans: b
Sol: The correct answer is option 2, which states that the vote value of an MP is calculated by dividing the total value of votes of all MLAs of all states and UTs by the total number of elected members of Parliament.
- This means that the vote value of an MP is determined by the number of MLAs in their respective states and UTs.
Other Related Points
- Option 1 is incorrect because it does not take into account the total number of elected members of Parliament.
- Option 3 is also incorrect because it assumes that all voters in the country have a say in the presidential election, which is not the case.
- Option 4 is incorrect because it only considers the members of Lok Sabha, whereas the presidential election involves both Lok Sabha and Rajya Sabha members.
Q31: Which of the following are principles of Panchsheel?
A. Mutual respect for each other's territorial integrity sovereignty
B. Mutual Non-aggression
C. Interference in each other's affairs
D. Equality and co-operation for mutual benefit
E. Equal Existence
Choose the most appropriate answer from the options given below :
(a) (A), (B), (C) only
(b) (A), (B), (D) only
(c) (B), (C), (D) only
(d) (C), (D), (E) only
Ans: b
Sol: The correct answer is option 2, which includes principles (A), (B), and (D) only.
- These principles of Panchsheel emphasize mutual respect for each other's territorial integrity and sovereignty, mutual non-aggression, and equality and cooperation for mutual benefit.
Other Related Points
- Option 1 includes principle (C), which is not a part of Panchsheel and goes against the principle of non-interference.
- Option 3 includes principle (B), which is a part of Panchsheel, but also includes principle (C), which is not.
- Option 4 includes principles (C) and (E), which are not a part of Panchsheel. Option 5 is not a valid option.
Q32: When did a Female Police Personnel from India took part in "first-ever peace keeping operations"?
(a) January 2007
(b) February 2008
(c) January 2006
(d) February 2009
Ans: a
Sol: The correct answer is option 1, January 2007.
In January 2007, a team of 105 Indian police personnel, including six women, were deployed to Liberia as part of the United Nations Mission in Liberia (UNMIL).
- This marked the first time that female police personnel from India had taken part in a peacekeeping operation.
- The women officers were trained in crowd control, human rights, and gender issues, and played a crucial role in building trust and confidence among the local population.
Other Related Points
- Option 2, February 2008, is incorrect as there is no record of any significant deployment of female police personnel from India in peacekeeping operations during this time.
- Option 3, January 2006, is also incorrect as there is no record of any significant deployment of female police personnel from India in peacekeeping operations during this time.
- Option 4, February 2009, is incorrect as there is no record of any significant deployment of female police personnel from India in peacekeeping operations during this time.
Q33: The 'extended neighbourhood' has become conceptual umbrella for Indian foreign policy. Which of the option given below does not come under purview of India's 'extended neighbourhood'?
(a) Suez Canal
(b) South China Sea
(c) Eastern Africa
(d) The Asia Pacific
Ans: c
Sol: The correct answer is option 3, Eastern Africa, as it is not considered a part of India's extended neighbourhood.
- India's extended neighbourhood includes regions such as the Asia Pacific, South China Sea, and the Suez Canal, which are of strategic importance to India's foreign policy.
- India has been actively engaging with these regions to enhance its economic, political, and security interests.
- For instance, India has been strengthening its ties with countries in the Asia Pacific region through initiatives such as the Act East Policy.
- Similarly, India has been closely monitoring developments in the South China Sea, which is a major trade route for India.
Overall, India's extended neighbourhood is a crucial component of its foreign policy, and the country has been working towards enhancing its presence and influence in these regions.
Q34: The foreign minister of India, S. Jaishankar has summerised India's approach to multilateral action as anchored in five 'S's. Choose the wrong option
(a) Samman (Respect)
(b) Samanjasya (Harmony)
(c) Samvad (Dialogue)
(d) Samriddi (Prosperity)
Ans: b
Sol: The correct answer is option 2, which is "Samanjasya" meaning harmony.
- This is because the other options, Samman (respect), Samvad (dialogue), Samriddi (prosperity), are all part of India's approach to multilateral action, but they are not the anchor of it.
- Harmony is the key to India's approach to multilateral action, as it emphasizes the importance of working together and finding common ground to achieve shared goals.
- Respect, dialogue, and prosperity are all important components of this approach, but they are not the central focus.
Q35: In which conference the decision to set up World Bank was taken?
(a) New York
(b) Bretton Woods
(c) Massachusetts
(d) Paris
Ans: b
Sol: The correct answer is option 2, Bretton Woods.
- The decision to set up the World Bank was taken at the United Nations Monetary and Financial Conference held in Bretton Woods, New Hampshire, in July 1944.
- The conference was attended by representatives of 44 countries and aimed to establish a new international monetary system after the end of World War II.
Other Related Points
- Option 1, New York, is incorrect as the decision to set up the World Bank was not taken in New York. However, the headquarters of the World Bank is located in Washington D.C., which is close to New York.
- Option 3, Massachusetts, is also incorrect as the decision to set up the World Bank was not taken in Massachusetts. However, Massachusetts is home to many prestigious universities and research institutions, including Harvard University, which has produced many economists who have contributed to the development of international finance and economics.
- Option 4, Paris, is incorrect as the decision to set up the World Bank was not taken in Paris. However, Paris is the capital of France, which is a member of the World Bank and has contributed to its development.
Overall, the correct answer is option 2, Bretton Woods, where the decision to set up the World Bank was taken.
Q36: In which year was the United Nations set up?
(a) 1944
(b) 1945
(c) 1946
(d) 1950
Ans: b
Sol: The correct answer is option 2, 1945.
- The United Nations was established on October 24, 1945, after the end of World War II.
- Its main purpose is to promote international cooperation and maintain peace and security among nations.
- The UN has 193 member states and is headquartered in New York City.
Other Related Points
- Option 1, 1944, is incorrect as this was the year the Bretton Woods Conference was held, which established the International Monetary Fund and the World Bank.
- Option 3, 1946, is incorrect as this was the year the International Court of Justice was established, but the UN was already in operation.
- Option 4, 1950, is incorrect as this was the year the Korean War began, which was a conflict that involved the UN, but the organization was already established.
Q37: Who coined the term 'containment'?
(a) George Kennan
(b) Alfred Sauvy
(c) George Marshall
(d) John Marshal
Ans: a
Sol: The correct answer is option 1, George Kennan.
- He was an American diplomat and historian who served as the US ambassador to the Soviet Union during the Cold War.
- In 1947, he wrote an article titled "The Sources of Soviet Conduct" under the pseudonym "X" in which he outlined the policy of containment.
- This policy aimed to prevent the spread of communism by containing it within its existing borders and using diplomatic, economic, and military means to weaken it.
Other Related Points
- Option 2, Alfred Sauvy, was a French demographer and economist who coined the term "Third World" in 1952 to describe the countries that were not aligned with either the capitalist First World or the communist Second World.
- Option 3, George Marshall, was an American military leader and statesman who served as the US Secretary of State and Secretary of Defense. He is best known for the Marshall Plan, which provided economic aid to Western Europe after World War II.
- Option 4, John Marshall, was an American politician and lawyer who served as the fourth Chief Justice of the United States from 1801 to 1835.
- Overall, George Kennan's concept of containment had a significant impact on US foreign policy during the Cold War and remains relevant today in discussions about how to deal with authoritarian regimes and threats to global security.
Q38: Who categorized India as "a responsible state with advanced nuclear technology"?
(a) President George W. Bush
(b) President Bill Clinton
(c) President Jimmy Carter
(d) President Barack Obama
Ans: a
Sol: The correct answer is option 1, President George W. Bush.
- In 2008, the United States signed a civil nuclear agreement with India, which recognized India as a responsible state with advanced nuclear technology.
- This agreement allowed India to engage in nuclear commerce with other countries, despite not being a signatory to the Nuclear Non-Proliferation Treaty.
- This was a significant shift in US-India relations, as previously the US had imposed sanctions on India for conducting nuclear tests in 1998.
Other Related Points
- Option 2, President Bill Clinton, imposed sanctions on India in response to their nuclear tests in 1998. This strained US-India relations for several years.
- Option 3, President Jimmy Carter, did not have a significant impact on US-India relations in regards to nuclear technology.
- Option 4, President Barack Obama, continued to support the US-India civil nuclear agreement and further strengthened US-India relations during his presidency.
Q39: Which of the following statements are true in Ecological Approach?
A. The term 'ecology' has Greek roots.
B. Ernst Haeckel coined the term 'ecology'.
C. Andrew Dobson makes a distinction between environmentalism and ecologism.
D. Green ideology debate is a 16th century debate.
Choose the most appropriate answer from the options given below:
(a) (A), (B), (C) only
(b) (B), (C) only
(c) (C), (D) only
(d) (A), (D) only
Ans: a
Sol: The correct answer is option 1, which means that all three statements (A, B, and C) are true in Ecological Approach.
- Statement A is true because the term "ecology" comes from the Greek words "oikos" (meaning "household" or "home") and "logos" (meaning "study" or "science").
- Statement B is also true because Ernst Haeckel, a German biologist, was the one who first used the term "ecology" in its modern sense in the late 19th century.
- Statement C is true because Andrew Dobson, a British ecologist, distinguishes between environmentalism (which focuses on protecting the environment for human benefit) and ecologism (which emphasizes the intrinsic value of nature and the need to protect it for its own sake).
Q40: Who among the following believes that the superstructure is relatively autonomous?
A. Louis Althusser
B. Gramsci
C. Nicos Poulantzas
D. Mao Zedong
Choose the correct answer from the options given below:
(a) (A) and (B) only
(b) (C) and (A) only
(c) (B), (C) and (D) only
(d) (B) only
Ans: c
Sol: The correct answer is option 3, which states that Gramsci, Poulantazs, and Mao Zedang all believe that the superstructure is relatively autonomous.
- Louis Althusser, on the other hand, believed that the superstructure is determined by the economic base and is not autonomous.
- Gramsci believed that the superstructure has a degree of autonomy and can influence the economic base.
- Poulantazs believed that the state and ideology have relative autonomy from the economic base. Mao Zedang believed that the superstructure can play a leading role in social change.
Q41: Which of the following statements are correct in terms of Machiavelli's 'The Prince'?
A. The book has been written with democratic spirit.
B. Dual morality has been established in this book.
C. The book is famous for anti secular approach.
D. The book has supported a citizen army.
Choose the most appropriate answer from the options given below:
(a) (B) and (D) only
(b) (A) and (C) only
(c) (C) and (D) only
(d) (B) and (C) only
Ans: a
Sol: The correct answer is option 1, which means that only statements (B) and (D) are correct in terms of Machiavelli's "The Prince.
- " Statement (B) is correct because Machiavelli establishes a dual morality in the book, where the ruler must be able to act immorally if it is necessary for the stability and success of the state.
- This is famously summarized in the phrase "the ends justify the means."
- Statement (D) is correct because Machiavelli supports the idea of a citizen army, where the people of the state are responsible for defending it rather than relying on mercenaries or professional soldiers. This is seen as a way to increase the loyalty and dedication of the army to the state.
- Statement (A) is incorrect because Machiavelli's book is not written with a democratic spirit. In fact, he argues that a strong and centralized monarchy is the best form of government.
- Statement (C) is also incorrect because Machiavelli's book is not anti-secular. While he does argue that religion can be a useful tool for the ruler to control the people, he does not reject the idea of a secular state.
Overall, Machiavelli's "The Prince" is a controversial and influential work that has been interpreted in many different ways over the centuries.
Q42: Which of the following sentences are correct in regards to John Locke?
A. He subscribes empiricism
B. He has written 'Essay Concerning Human Understanding'.
C. He believes in limited state.
D. He is the propounder of popular sovereignty.
Choose the most appropriate answer from the options given below:
(a) (A), (B), (C) only
(b) (C), (D) only
(c) (A), (D) only
(d) (B), (C) only
Ans: a
Sol: The correct answer is option 1, which means that sentences A, B, and C are correct in regards to John Locke.
- Sentence A is correct because Locke is known for his empiricist philosophy, which emphasizes the importance of experience and observation in acquiring knowledge.
- Sentence B is correct because Locke's "Essay Concerning Human Understanding" is one of his most famous works, in which he explores the nature of human knowledge and understanding.
- Sentence C is correct because Locke believed in the concept of limited government, which means that the power of the state should be restricted in order to protect individual rights and freedoms.
- Sentence D is incorrect because Locke is not typically associated with the concept of popular sovereignty, which is the idea that the people are the ultimate source of political power.
- This concept is more closely associated with thinkers like Jean-Jacques Rousseau and Thomas Paine. Overall, John Locke was a highly influential philosopher who made significant contributions to the fields of epistemology, political philosophy, and economics.
- His ideas about the importance of individual rights and limited government continue to be influential today.
Q43: Which of the following are correct in reference to Tagore's Philosophy?
A. Tagore was an exponent of 'true freedom'.
B. Tagore appeared to be a seeker of eternal truth.
C. Tagore supported creative unity amongst diversity.
D. Tagore supported aggressive nationalism.
Choose the most appropriate answer from the options given below:
(a) (A), (B), (C) only
(b) (A), (B), (D) only
(c) (B), (C), (D) only
(d) (A), (C), (B), (D)
Ans: a
Sol: The correct answer is option 1, which states that Tagore was an exponent of 'true freedom', appeared to be a seeker of eternal truth, and supported creative unity amongst diversity.
- These three statements accurately reflect Tagore's philosophy, which emphasized the importance of individual freedom, the search for universal truth, and the celebration of diversity and cultural exchange.
Other Related Points
- Option 2, which includes (B), (C), and (D) only, is incorrect because it leaves out the important statement that Tagore was an exponent of 'true freedom'.
- Option 3, which includes (A), (C), and (D) only, is incorrect because it includes the statement that Tagore supported aggressive nationalism, which is not true.
- Option 4, which includes all four statements, is incorrect because it includes the false statement that Tagore supported aggressive nationalism.
Q44: What is the purpose of Danda Niti according to Kautilya?
A. Acquisition of unacquired
B. Preservation of the acquired
C. Maintenance of law and order within the state
D. Augmentation of the preserved
E. Fair distribution of the augmented
Choose the most appropriate answer from the options given below:
(a) (A), (B), (C), (D) only
(b) (B), (C), (D), (E) only
(c) (A), (C), (D), (E) only
(d) (A), (B), (D), (E) only
Ans: d
Sol: The correct answer is option 4, which includes the purposes of Danda Niti as acquisition of unacquired, preservation of the acquired, augmentation of the preserved, and fair distribution of the augmented.
- Option A only includes the acquisition of unacquired, which is only one aspect of Danda Niti.
- Option B includes the preservation of the acquired, which is important but does not encompass all the purposes of Danda Niti.
- Option C includes the maintenance of law and order within the state, which is a related but separate concept from Danda Niti.
- Option E includes the fair distribution of the augmented, which is important but again does not encompass all the purposes of Danda Niti. Danda Niti is a concept in ancient Indian political philosophy, particularly in the works of Kautilya.
- It refers to the use of punishment and force as a means of maintaining order and achieving political goals.
- The purposes of Danda Niti include not only the acquisition and preservation of power, but also the fair distribution of resources and benefits to the people.
Q45: Which of the following statements is/are correct regarding the powers of Speaker of Lok Sabha?
A. The Anti Defection Law is applicable on the Speaker in case he decides to quit his own party.
B. The Speaker can not preside while a resolution for his removal from office is under consideration.
C. Speaker can vote in the house on any matters of his removal from the office.
Choose the correct answer from the options given below:
(a) (A), (B) and (C)
(b) (B) and (C) only
(c) (A) and (C) only
(d) (A) and (B) only
Ans: b
Sol: The correct answer is option 2, which means that only statements (B) and (C) are correct regarding the powers of Speaker of Lok Sabha.
- Statement (A) is incorrect because the Anti Defection Law is not applicable on the Speaker if he decides to quit his own party.
- However, if the Speaker is elected from a particular party and later decides to join another party, he may be disqualified under the Anti Defection Law.
- Statement (B) is correct because the Speaker cannot preside over the house while a resolution for his removal from office is under consideration. In such a case, the Deputy Speaker or any other member of the house presides over the proceedings.
- Statement (C) is correct because the Speaker can vote in the house on any matters of his removal from the office. However, he cannot use his casting vote in such a situation.
- Overall, the Speaker of Lok Sabha has several powers and responsibilities, including maintaining order and decorum in the house, deciding on the admissibility of questions and motions, and presiding over the proceedings. The Speaker also has the power to disqualify members under certain circumstances and can use his casting vote in case of a tie.
Q46: Which of the following statements are correct regarding preamble of the Indian Constitution?
A. In the Berubari Case, Supreme Court said that the preamble is not part of the Constitution.
B. In the Keshavananda Bharati case, the Supreme Court said that preamble is the part of the Constitution.
C. In L.I.C. of India Case, Supreme Court said that preamble is not part of the Constitution.
Choose the correct answer from the options given below:
(a) (A), (B) and (C)
(b) (B) and (C) only
(c) (A) and (B) only
(d) (A) and (C) only
Ans: c
Sol: The correct answer is option 3, which is (A) and (B) only.
- In the Berubari Case, the Supreme Court did say that the preamble is not a part of the Constitution.
- However, in the Keshavananda Bharati case, the Supreme Court reversed its earlier decision and held that the preamble is a part of the Constitution.
- Therefore, option B is correct. Option C is incorrect as the Supreme Court has not said in the L.I.C. of India Case that the preamble is not a part of the Constitution.
- Option A and D are incorrect as they do not include the correct statement about the Keshavananda Bharati case.
Q47: Which of the following are functions of a political system?
A. Political Socialisation
B. Political Decay
C. Political Recruitment
D. Political Revolutions
Choose the most appropriate answer from the options given below:
(a) (A) and (B) only
(b) (B) and (C) only
(c) (A) and (C) only
(d) (B) and (D) only
Ans: c
Sol: The correct answer is option 3, which states that political socialisation and political recruitment are functions of a political system.
- Political socialisation refers to the process by which individuals learn about the political system and its values, while political recruitment refers to the process by which individuals are selected for political positions.
Other Related Points
- Option 1, which states that political socialisation and political decay are functions of a political system, is incorrect because political decay is not a function of a political system. Political decay refers to the decline of a political system and is a negative outcome, not a function.
- Option 2, which states that political decay and political revolutions are functions of a political system, is also incorrect because political revolutions are not a function of a political system. Political revolutions are a response to a political system that is perceived as unjust or oppressive, but they are not a necessary or desirable function of a political system.
- Option 4, which states that political decay and political revolutions are functions of a political system, is also incorrect for the same reasons as option 2.
Q48: Which of the below mentioned factors lead to revolution in a political system?
A. Severe class divisions
B. Role of intellectuals in mass mobilisation
C. Social disequilibrium
D. Strong desire for peace
E. Incremental changes
Choose the most appropriate answer from the options given below:
(a) (A), (B) and (E) only
(b) (B), (C) and (D) only
(c) (A), (B) and (C) only
(d) (A), (B), (C), (D) and (E)
Ans: c
Sol: The correct answer is option 3, which includes severe class divisions, the role of intellectuals in mass mobilisation, and social disequilibrium as factors that lead to revolution in a political system.
- These factors create a sense of discontent and frustration among the masses, leading to a desire for change and a willingness to take action.
- Incremental changes and a strong desire for peace are not typically associated with revolution, as they suggest a more gradual and peaceful approach to political change.
Other Related Points
- Option 1 includes severe class divisions, the role of intellectuals in mass mobilisation, and incremental changes, but it does not include social disequilibrium, which is an important factor in creating the conditions for revolution.
- Option 2 includes the role of intellectuals in mass mobilisation, social disequilibrium, and a strong desire for peace, but it does not include severe class divisions, which are often a key driver of revolutionary movements.
- Option 4 includes all of the factors mentioned, but it is not necessarily the case that all of these factors must be present for a revolution to occur. In some cases, one or two of these factors may be sufficient to spark a revolutionary movement.
Q49: According to the article 26 of the constitution of India, every religious denomination shall have the right :
A. to establish and maintain institutions for religious and charitable purpose.
B. to manage its own affairs in matters of religion.
C. to own and acquire movable property.
D. to manage social affairs.
E. to own and acquire immovable property.
Choose the most appropriate answer from the options given below:
(a) (A), (B), (C), (D) only
(b) (B), (C), (D), (E) only
(c) (A), (B), (C), (E) only
(d) (A), (C), (D), (E) only
Ans: c
Sol: The correct answer is option 3, which includes the statements A, B, C, and E.
- This is because a religious trust or institution is primarily established to serve religious and charitable purposes, manage its own affairs in matters of religion, own and acquire movable and immovable property, and maintain its social affairs.
- These are the essential functions of a religious trust or institution.
Other Related Points
- Option 1 includes statements A, B, C, and D, which is incorrect because managing social affairs is not a primary function of a religious trust or institution.
- Option 2 includes statements B, C, D, and E, which is also incorrect because it does not include the statement about establishing and maintaining institutions for religious and charitable purposes.
- Option 4 includes statements A, C, D, and E, which is incorrect because it does not include the statement about managing its own affairs in matters of religion.
Overall, a religious trust or institution is established to serve religious and charitable purposes, manage its own affairs in matters of religion, own and acquire movable and immovable property, and maintain its social affairs.
Q50: According to Ferral Heady, the comparative public administration addresses which of the following concerns?
A. The search of theory
B. The urge of practical application
C. Contribution to the border field of comparative politics
D. The comparative analysis of ongoing problems of Public Administration
Choose the most appropriate answer from the options given below :
(a) (A), (B), (C) and (D)
(b) (A), (B) and (C) only
(c) (B), (C) and (D) only
(d) (A), (C) and (D) only
Ans: a
Sol: The correct answer is option 1, which means that comparative public administration addresses all of the concerns mentioned in the options (A), (B), (C), and (D).
- Ferrel Heady's approach to comparative public administration emphasizes the need for both theoretical and practical considerations, as well as the contribution to the broader field of comparative politics.
- Additionally, Heady's approach involves the comparative analysis of ongoing problems in public administration across different countries and contexts.
Other Related Points
- Option 2, which includes (A), (B), and (C) only, is incorrect because it excludes the comparative analysis of ongoing problems in public administration, which is a key concern of comparative public administration according to Heady.
- Option 3, which includes (B), (C), and (D) only, is incorrect because it excludes the search for theory, which is also an important concern of comparative public administration according to Heady.
- Option 4, which includes (A), (C), and (D) only, is incorrect because it excludes the urge for practical application, which is also a key concern of comparative public administration according to Heady.
Q51: Which of the following are not the features of Cold War?
A. There was nuclear arms race.
B. There was a propaganda war.
C. There was economic blockade against the non-aligned countries.
D. The third world countries were directly involved in it.
Choose the correct answer from the options given below:
(a) (A) and (B) only
(b) (B) and (C) only
(c) (C) and (D) only
(d) (B) and (D) only
Ans: c
Sol: The correct answer is option 3, which states that economic blockade against non-aligned countries and direct involvement of third world countries were not features of the Cold War.
- Option A is correct as the Cold War was characterized by a nuclear arms race between the United States and the Soviet Union.
- Option B is also correct as both sides engaged in a propaganda war to promote their ideologies and gain support from other countries.
- Option C is incorrect as there was no economic blockade against non-aligned countries during the Cold War. However, both the United States and the Soviet Union did provide economic aid to countries that aligned with their respective ideologies.
- Option D is incorrect as third world countries were indirectly involved in the Cold War through proxy wars and support from the superpowers. The United States and the Soviet Union often provided military and economic aid to third world countries in order to gain influence and support in the region.
Overall, the Cold War was a period of political and military tension between the United States and the Soviet Union, characterized by a nuclear arms race, propaganda war, and indirect involvement of third world countries.
Q52: Given below are two statements: One is labelled as Assertion (A) and the other is labelled as Reason (R).
Assertion (A): Ever since the end of World War II, international environment has been dominated by the American relations with the Soviet Union characterized as Cold War.
Reason (R): The Cold War coloured not only American and American aligned nations' relations with the Soviet Union and the latter's allies but relations of the U.S. and its allies with other non-aligned nations, as well.
In the light of the above statements, choose the correct answer from the options given below:
(a) Both (A) and (R) are true and (R) is the correct explanation of (A)
(b) Both (A) and (R) are true but (R) is NOT the correct explanation of (A)
(c) (A) is true but (R) is false
(d) (A) is false but (R) is true
Ans: a
Sol: The correct answer is Both (A) and (R) are true and (R) is the correct explanation of (A).
- Assertion (A) is true because the Cold War was a period of geopolitical tension between the United States and the Soviet Union and their respective allies, the Western Bloc and the Eastern Bloc, which began following World War II.
- Reason (R) is also true because the Cold War had a profound impact on the international environment, shaping the foreign policy of many countries and leading to the formation of alliances and the development of new weapons. The Cold War also had a significant impact on the domestic politics of many countries, as governments were forced to choose sides in the conflict.
- Furthermore, Reason (R) is the correct explanation of Assertion (A) because it explains how the Cold War colored not only American and American aligned nations' relations with the Soviet Union and the latter's allies but relations of the U.S. and its allies with other non-aligned nations, as well.
In conclusion, both Assertion (A) and Reason (R) are true and Reason (R) is the correct explanation of Assertion (A).
Q53: Which of the following is/are true about amendment of RTI Act in 2019?
A. From a fixed term of five years, now the Commissioner holds office at the pleasure of Government.
B. Now, there can be more than 10 Central Information Commissioners.
C. Now, the salaries and allowances of Commissioner are also decided by the Government.
D. Now, the age of retirement of Chief Information Commissioner will be 62 years.
Choose the correct answer from the options given below:
(a) (C) only
(b) (A) and (B) only
(c) (A) and (C) only
(d) (A) and (D) only
Ans: c
Sol: The correct answer is option 3, which means that only statements A and C are true about the amendment of RTI Act in 2019.
- Statement A is true because the amendment removed the fixed term of five years for Information Commissioners and now they hold office at the pleasure of the Government.
- This means that the Government can remove them from their position at any time without any specific reason.
- Statement C is also true because the amendment gave the Government the power to decide the salaries and allowances of Information Commissioners. Earlier, these were decided by an independent committee.
- Statement B is false because the amendment did not change the maximum number of Central Information Commissioners, which is still 10.
- Statement D is also false because the amendment did not change the age of retirement of Chief Information Commissioner, which is still 65 years.
Overall, the 2019 amendment to the RTI Act has been criticized by many as it reduces the autonomy and independence of Information Commissioners and gives more power to the Government.
Q54: Given below are two statements: One is labelled as Assertion (A) and the other is labelled as Reason (R).
Assertion (A): In our time, history is that which transforms documents in to monuments.
Reason (R): The archaeology of knowledge discovers the intrinsic description of the monuments.
In the light of the above statements, choose the most appropriate answer from the options given below:
(a) Both (A) and (R) are correct and (R) is the correct explanation of (A)
(b) Both (A) and (R) are correct but (R) is NOT the correct explanation of (A)
(c) (A) is correct but (R) is not correct
(d) (A) is not correct but (R) is correct
Ans: a
Sol: The correct answer is option 1: Both (A) and (R) are correct and (R) is the correct explanation of (A).
- Explanation: (A) states that in our time, history is that which transforms documents into monuments. This means that history is not just about recording events, but also about creating lasting symbols and representations of those events.
- (R) states that the archaeology of knowledge discovers the intrinsic description of the monuments. This means that by studying the physical artifacts and structures that represent historical events, we can gain a deeper understanding of the knowledge and ideas that were present at the time.
- Therefore, (R) is the correct explanation of (A), as it shows how the transformation of documents into monuments is related to the discovery of knowledge through archaeology.
Other Related Points
- Option 2 is incorrect because it acknowledges that both (A) and (R) are correct, but incorrectly states that (R) is not the correct explanation of (A).
- Option 3 is incorrect because it states that (A) is correct but (R) is not correct, which is not true.
- Option 4 is incorrect because it states that (R) is correct but (A) is not correct, which is also not true.
Overall, the correct answer is option 1 because both (A) and (R) are correct and (R) is the correct explanation of (A).
Q55: Given below are two statements: One is labelled as Assertion (A) and the other is labelled as Reason (R).
Assertion (A): Inner contradictions of capitalism will destroy capitalism.
Reason (R): Exploitation of surplus value is the biggest contradiction of capitalism.
In the light of the above statements, choose the correct answer from the options given below:
(a) Both (A) and (R) are true and (R) is the correct explanation of (A)
(b) Both (A) and (R) are true but (R) is NOT the correct explanation of (A)
(c) (A) is true but (R) is false
(d) (A) is false but (R) is true
Ans: a
Sol: The correct answer is option 1 - Both (A) and (R) are true and (R) is the correct explanation of (A).
This is because the statement in (A) is true - capitalism does have inner contradictions that will eventually lead to its downfall.
The statement in (R) is also true - the exploitation of surplus value, or the difference between the value of a worker's labor and the wages they are paid, is one of the biggest contradictions of capitalism.
This exploitation leads to inequality and ultimately undermines the stability of the system. Therefore, (R) is the correct explanation of (A).
Option 2 - Both (A) and (R) are true but (R) is NOT the correct explanation of (A) - is incorrect because (R) is indeed the correct explanation of (A).
Option 3 - (A) is true but (R) is false - is incorrect because (R) is also true.
Option 4 - (A) is false but (R) is true - is incorrect because (A) is actually true.
Q56: Given below are two statements: One is labelled as Assertion (A) and the other is labelled as Reason (R).
Assertion (A): Economically, the framework of the 'Sarvodaya' order would seek to establish a balanced and equitable economic setup in the country.
Reason (R): J.P.(Jai Prakash) was sure to afford the first place to agricultural activities in the economic life of the people.
In the light of the above statements, choose the most appropriate answer from the options given below:
(a) Both (A) and (R) are correct and (R) is the correct explanation of (A)
(b) Both (A) and (R) are correct but (R) is NOT the correct explanation of (A)
(c) (A) is correct but (R) is not correct
(d) (A) is not correct but (R) is correct
Ans: b
Sol: The correct answer is option 2 - Both (A) and (R) are correct but (R) is NOT the correct explanation of (A). Statement (A) states that the framework of the 'Sarvodaya' order aims to establish a balanced and equitable economic setup in the country. This statement is correct. Statement (R) states that J.P. gives priority to agricultural activities in the economic life of the people. This statement is also correct. However, it is not a direct explanation of the first statement. The two statements are related, but statement (R) does not provide a direct explanation of statement (A).
- Option 1 - Both (A) and (R) are correct and (R) is the correct explanation of (A) - is incorrect because statement (R) does not provide a direct explanation of statement (A).
- Option 3 - (A) is correct but (R) is not correct - is incorrect because statement (R) is correct.
- Option 4 - (A) is not correct but (R) is correct - is incorrect because statement (A) is correct.
- Option 5 - The correct answer is option 2 - has already been explained above.
Q57: Which of the following are strengths of Globalization?
A. Promotes free trade of goods and services among the nations.
B. Strengthens the market economies of the developing world.
C. Promotes socio-economic interactions between the countries.
D. Increases economic disparities between rich and poor.
E. It does not promote human and cultural interaction between the nations.
Choose the most appropriate answer from the options given below:
(a) (A), (B) and (C) only
(b) (B), (C) and (D) only
(c) (C), (D) and (E) only
(d) (A), (D) and (E) only
Ans: a
Sol: The correct answer is option 1, which includes strengths of globalization such as promoting free trade of goods and services among nations, strengthening the market economies of the developing world, and promoting socio-economic interactions between countries.
- These strengths can lead to increased economic growth and development, as well as greater cultural exchange and understanding.
Other Related Points
- Option 2 includes strengths such as strengthening market economies and promoting socio-economic interactions, but also includes the negative aspect of increasing economic disparities between rich and poor.
- Option 3 includes the positive aspect of promoting socio-economic interactions, but also includes the negative aspect of increasing economic disparities and the false statement that globalization does not promote human and cultural interaction.
- Option 4 includes the positive aspect of promoting free trade and the negative aspect of increasing economic disparities and lack of human and cultural interaction.
Q58: Which is/are not true about Public Interest Litigation (PIL)?
A. Only registered Civil Society Organisation can file a PIL.
B. PIL can be filed in Supreme Court, under Article 32.
C. PIL can be filed in High Courts under Article 226.
D. There is no court fees for filing PIL.
Choose the most appropriate answer from the options given below:
(a) (B) and (C) only
(b) (A) only
(c) (D) only
(d) (A) and (D) only
Ans: d
Sol: The correct answer is option 4, which means that statements A and D are not true about Public Interest Litigation (PIL).
- In reality, any citizen can file a PIL, and there may be court fees associated with it.
- PIL is a legal tool that allows individuals or organizations to seek justice on behalf of the public or a marginalized group.
- It is often used to address issues related to human rights, environmental protection, and corruption.
- PIL can be filed in both the Supreme Court (under Article 32) and High Courts (under Article 226) of India.
Q59: What are the core values of Indian Foreign Policy?
A. Blend of idealism and realism
B. Value of Tolerance
C. Anti-imperialism and anti-colonialism
D. Territorial Expansion
E. Neighbourhood First Diplomacy
Choose the correct answer from the options given below:
(a) (A), (B), (C), (E) only
(b) (A), (B), (D) only
(c) (B), (C), (D) only
(d) (A), (B), (D), (E) only
Ans: a
Sol: The correct answer is option 1,
- In the decades following independence, Indian foreign policy has been characterized by a blend of idealism and realism (Option A). This has involved both adherence to certain moral and ethical principles, as well as pragmatic considerations based on national interest. For instance, India has consistently advocated for global peace, disarmament, and a more equitable international order, while also engaging in strategic alliances and partnerships based on its security and economic needs .
- Indian foreign policy also reflects a deep-seated commitment to the value of tolerance (Option B). This can be seen in India's respectful engagement with countries of various ideological and political dispensations, its historical role as a cultural melting pot, as well as its strong advocacy for mutual respect and peaceful coexistence among nations .
- Anti-imperialism and anti-colonialism (Option C) remain other crucial pillars of Indian foreign policy, shaped by India's own historical experience as a British colony. India has steadfastly supported anti-colonial struggles worldwide, and has condemned all forms of imperialism and external interference in the internal affairs of sovereign nations .
- In terms of 'Neighbourhood First' diplomacy (Option E), this policy was more recently articulated by Prime Minister Narendra Modi, with the goal of strengthening ties with neighbouring countries and promoting regional integration. This policy reflects the recognition that a peaceful and stable neighbourhood is crucial to India's own security and development .
- However, territorial expansion (Option D) is not an objective of Indian foreign policy. India, abiding by the principles of the United Nations Charter, respects the sovereignty and territorial integrity of all countries. Hence, this option doesn't form a part of the core values of Indian foreign policy. Therefore, options A, B, C, and E correctly represent the core values of Indian foreign policy.
Q60: Which of the following is correct regarding the classical theory of administration?
A. It played a notable role in rationalizing and even stimulating production.
B. It formulated a set of concepts in administration.
C. It was the first to propound the idea that administration is a separate activity.
Choose the most appropriate answer from the options given below:
(a) (A) and (B) only
(b) (B) and (C) only
(c) (A) and (C) only
(d) (A), (B) and (C)
Ans: d
Sol: The correct answer is option 4, which states that all three statements are correct regarding the classical theory of administration.
- Option A is correct because the classical theory emphasized efficiency and productivity in organizations, and sought to rationalize and streamline production processes.
- Option B is correct because the classical theory developed a set of concepts and principles in administration, such as the division of labor, hierarchy, and specialization.
- Option C is correct because the classical theory was the first to propose that administration is a distinct and separate activity from other functions in an organization, such as management or production.
Other Related Points
- Option 1 is incorrect because it only includes statements A and B, but leaves out the important idea that administration is a separate activity.
- Option 2 is incorrect because it only includes statements B and C, but ignores the emphasis on productivity and rationalization in the classical theory.
- Option 3 is incorrect because it only includes statements A and C, but overlooks the development of concepts and principles in administration that were central to the classical theory.
Q61: Given below are two statements: One is labelled as Assertion (A) and the other is labelled as Reason (R).
Assertion (A): Political culture is a set of beliefs and sentiments about politics as embodied in the verbal and non-verbal behaviour of the people.
Reason (R): The typological and morphological models of political culture denote that there can be as many political cultures as the dynamics of political development in political systems.
In the light of the above statements, choose the most appropriate answer from the options given below :
(a) Both (A) and (R) are correct and (R) is the correct explanation of (A)
(b) Both (A) and (R) are correct but (R) is NOT the correct explanation of (A)
(c) (A) is correct but (R) is not correct
(d) (A) is not correct but (R) is correct
Ans: b
Sol: The correct answer is option 2: Both (A) and (R) are correct but (R) is NOT the correct explanation of (A).
- The statement in (A) is correct as political culture refers to the beliefs and attitudes of people towards politics, which can be observed through their behavior.
- The statement in (R) is also correct as typological and morphological models of political culture suggest that there can be different political cultures based on the political development of a system.
- However, (R) is not the correct explanation of (A) as it does not directly explain or clarify the statement in (A). The two statements are related but not dependent on each other.
Other Related Points
- Option 1 is incorrect as (R) is not the correct explanation of (A).
- Option 3 is incorrect as (A) is correct.
- Option 4 is incorrect as (R) is correct.
Q62: Given below are two statements: One is labelled as Assertion (A) and the other is labelled as Reason (R).
Assertion (A): Robert Dahl has suggested for cross cultural studies in Public Administration.
Reason (R): He believed that the Public Administration can escape the effects of political, cultural and environmental system in which it develops.
In the light of the above statements, choose the most appropriate answer from the options given below:
(a) Both (A) and (R) are correct and (R) is the correct explanation of (A)
(b) Both (A) and (R) are correct but (R) is NOT the correct explanation of (A)
(c) (A) is correct but (R) is not correct
(d) (A) is not correct but (R) is correct
Ans: c
Sol: The correct response is (A) is correct but (R) is not correct.
- Assertion (A) is correct: Robert Dahl, a prominent political scientist, strongly advocated for the importance of cross-cultural studies in Public Administration.
- He believed that studying public administration across different cultures would help improve our understanding of the field and identify general principles that transcend national boundaries.
- Reason (R) is incorrect: However, Dahl did not believe that public administration could escape the effects of political, cultural, and environmental factors.
- He understood that these factors significantly influence how public administration functions in different contexts. Studying these influences was what he emphasized in his call for cross-cultural research.
Other Related Points
Robert Dahl impacted public administration by:
- Pushing for cross-cultural research: Seeing beyond national boundaries to learn from diverse governance models.
- Challenging neutrality: Emphasizing the political and social context shaping how administration works.
- Promoting ethical decision-making: Highlighting the importance of values and fairness in governing practices.
Q63: Given below are two statements :
Statement (I): Rights essentially belong to the sphere of conflicting claims between individual and the state.
Statement (II): Any political theory which holds that an individual cannot have certain rights 'against the state' is no theory of rights.
In the light of the above statements, choose the correct answer from the options given below:
(a) Both Statement (I) and Statement (II) are true
(b) Both Statement (I) and Statement (II) are false
(c) Statement (I) is true but Statement (II) is false
(d) Statement (I) is false but Statement (II) is true
Ans: a
Sol: The correct answer is option 1, which means that both Statement (I) and Statement (II) are true.
- Statement (I) is true because rights are indeed a matter of conflicting claims between individuals and the state. Individuals have certain rights that they believe should be protected by the state, while the state may have its own interests and priorities that may conflict with those rights. This tension between individual rights and state power is a fundamental aspect of political theory.
- Statement (II) is also true because any political theory that denies individuals the right to have certain rights against the state cannot be considered a theory of rights.
- In other words, if a political theory does not recognize that individuals have rights that the state must respect and protect, then it cannot be considered a theory of rights.
Other Related Points
- Option 2, which states that both statements are false, is incorrect because both statements are actually true.
- Option 3, which states that Statement (I) is true but Statement (II) is false, is also incorrect because Statement (II) is true.
- Option 4, which states that Statement (I) is false but Statement (II) is true, is also incorrect because Statement (I) is true.
Q64: Given below are two statements :
Statement (I): Problems of global environmental change could not have existed without modern technologies and population densities.
Statement (II): Problems of climate change are difficult to address, because they involve both intra and inter-generational collective
action problems.
In the light of the above statements, choose the correct answer from the options given below:
(a) Both Statement (I) and Statement (II) are true
(b) Both Statement (I) and Statement (II) are false
(c) Statement (I) is true but Statement (II) is false
(d) Statement (I) is false but Statement (II) is true
Ans: a
Sol: The correct answer is option 1 - Both Statement (I) and Statement (II) are true.
- Statement (I) is true because modern technologies and population densities have contributed to global environmental problems such as climate change, pollution, and resource depletion. Without these factors, these problems would not have reached the scale and severity that they have today.
- Statement (II) is also true because addressing climate change requires collective action from both current and future generations.
- This involves making difficult decisions about reducing greenhouse gas emissions, transitioning to renewable energy sources, and adapting to the impacts of climate change.
- These decisions require cooperation and coordination at both the national and international levels.
Other Related Points
- Option 2 - Both Statement (I) and Statement (II) are false - is incorrect because both statements are true.
- Option 3 - Statement (I) is true but Statement (II) is false - is incorrect because Statement (II) is also true.
- Option 4 - Statement (I) is false but Statement (II) is true - is incorrect because Statement (I) is true.
Q65: Given below are two statements :
Statement (I): Nehru favoured a socialistic pattern of society.
Statement (II): Nehru's commitment to civil liberties prevented him from accepting the idea of 'dictatorship of the proletariat'.
In the light of the above statements, choose the correct answer from the options given below:
(a) Both Statement (I) and Statement (II) are true
(b) Both Statement (I) and Statement (II) are false
(c) Statement (I) is true but Statement (II) is false
(d) Statement (I) is false but Statement (II) is true
Ans: a
Sol: The correct answer is option 1 - Both Statement (I) and Statement (II) are true.
- Statement (I) is true as Nehru did favour a socialistic pattern of society and implemented policies such as land reforms, nationalization of industries, and the establishment of public sector enterprises.
- Statement (II) is also true as Nehru was committed to civil liberties and democracy, and did not support the idea of a dictatorship of the proletariat, which is a concept in Marxist theory that advocates for the working class to seize control of the government and establish a socialist state.
Other Related Points
- Option 2 - Both Statement (I) and Statement (II) are false - is incorrect as both statements are true.
- Option 3 - Statement (I) is true but Statement (II) is false - is incorrect as Statement (II) is also true.
- Option 4 - Statement (I) is false but Statement (II) is true - is incorrect as Statement (I) is true.
Q66: Given below are two statements :
Statement (I): Party system provides normal means of fighting political battles.
Statement (II): One of the aims of the party system is to replace the traditional elites with new elites.
In the light of the above statements, choose the most appropriate answer from the options given below:
(a) Both Statement (I) and Statement (II) are correct
(b) Both Statement (I) and Statement (II) are incorrect
(c) Statement (I) is correct but Statement (II) is incorrect
(d) Statement (I) is incorrect but Statement (II) is correct
Ans: a
Sol: The correct answer is option 1 - Both Statement (I) and Statement (II) are correct.
- Statement (I) is correct because the party system is a normal means of fighting political battles. Political parties provide a platform for individuals to contest elections and put forth their political ideologies and agendas. Through the party system, individuals can engage in healthy political competition and work towards achieving their goals.
- Statement (II) is also correct because one of the aims of the party system is to replace the traditional elites with new elites. Political parties often represent the interests of certain sections of society and work towards promoting their agenda. In doing so, they may challenge the existing power structures and replace the traditional elites with new ones who represent the interests of their party and its supporters.
Other Related Points
- Option 2 - Both Statement (I) and Statement (II) are incorrect - is incorrect because both statements are actually correct.
- Option 3 - Statement (I) is correct but Statement (II) is incorrect - is incorrect because Statement (II) is also correct.
- Option 4 - Statement (I) is incorrect but Statement (II) is correct - is incorrect because Statement (I) is actually correct.
Q67: Given below are two statements:
Statement (I): Exit poll is a post-election poll which is conducted when an elector walks out after casting his/her vote.
Statement (II): Opinion poll, is a kind of research survey which is conducted to find out the public opinion after elections.
In the light of the above statements, choose the correct answer from the options given below:
(a) Both Statement (I) and Statement (II) are true
(b) Both Statement (I) and Statement (II) are false
(c) Statement (I) is true but Statement (II) is false
(d) Statement (I) is false but Statement (II) is true
Ans: c
Sol: The correct answer is option 3 - Statement (I) is true but Statement (II) is false.
- Exit polls are conducted on the day of the election, as voters leave the polling station, to predict the outcome of the election.
- On the other hand, opinion polls are conducted before or after the election to gauge public opinion on various issues.
Other Related Points
- Option 1 - Both statements are true, which is incorrect as only Statement (I) is true.
- Option 2 - Both statements are false, which is incorrect as Statement (I) is true.
- Option 4 - Statement (I) is false but Statement (II) is true, which is incorrect as Statement (I) is true.
Q68: Given below are two statements: Consider the following statements regarding the Vice President of India.
Statement (I): He serves for a five year term. But can continue to be in office, irrespective of the expiry of the term, until the successor assumes office.
Statement (II): The constitution is silent on who performs the duties of the Vice-President, when a vacancy occurs in the office before expiry of his term.
In the light of the above statements, choose the most appropriate answer from the options given below:
(a) Both Statement (I) and Statement (II) are correct
(b) Both Statement (I) and Statement (II) are incorrect
(c) Statement (I) is correct but Statement (II) is incorrect
(d) Statement (I) is incorrect but Statement (II) is correct
Ans: a
Sol: The correct answer is option 1 - Both Statement (I) and Statement (II) are correct.
- "Statement (I) is correct per Article 67, which states the Vice-President serves a five-year term but may continue until a successor assumes office.
- Statement (II) is correct, as the Constitution does not specify who performs the Vice-President’s duties during a vacancy before term expiry.
Other Related Points
- Option 2 - Both Statement (I) and Statement (II) are incorrect - is not correct as both statements are factually correct.
- Option 3 - Statement (I) is correct but Statement (II) is incorrect - is not correct as both statements are factually correct.
- Option 4 - Statement (I) is incorrect but Statement (II) is correct - is not correct as Statement (I) is factually correct.
Q69: Given below are two statements:
Statement (I): The policy agenda is the sum of all political demands that a number of individuals or groups have tried to get onto policy agenda.
Statement (II): The policy agenda is composed of the demands that policy makers have agreed to consider.
In the light of the above statements, choose the most appropriate answer from the options given below:
(a) Both Statement (I) and Statement (II) are correct
(b) Both Statement (I) and Statement (II) are incorrect
(c) Statement (I) is correct but Statement (II) is incorrect
(d) Statement (I) is incorrect but Statement (II) is correct
Ans: d
Sol: The correct answer is option 4 - Statement (I) is incorrect but Statement (II) is correct.
- The policy agenda is not simply the sum of all political demands, as not all demands are considered by policy makers.
- Rather, the policy agenda is composed of the demands that policy makers have agreed to consider. This means that some demands may be excluded from the policy agenda, while others may be prioritized or given more attention.
Other Related Points
- Option 1 - Both Statement (I) and Statement (II) are correct - is incorrect because Statement (I) is not entirely accurate.
- Option 2 - Both Statement (I) and Statement (II) are incorrect - is incorrect because Statement (II) is actually correct.
- Option 3 - Statement (I) is correct but Statement (II) is incorrect - is incorrect because Statement (I) is not entirely accurate. It is important to understand the concept of the policy agenda in order to analyze and understand the political process and how certain issues are addressed or ignored by policy makers.
Q70: Given below are two statements:
Statement (I): Both IMF and IBRD have headquarters in Washington.
Statement (II): India's vote share in IMF is 10%.
In the light of the above statements, choose the most appropriate answer from the options given below:
(a) Both Statement (I) and Statement (II) are correct
(b) Both Statement (I) and Statement (II) are incorrect
(c) Statement (I) is correct but Statement (II) is incorrect
(d) Statement (I) is incorrect but Statement (II) is correct
Ans: c
Sol: The correct answer is option 3, which means that Statement (I) is correct but Statement (II) is incorrect.
- This means that both the International Monetary Fund (IMF) and the International Bank for Reconstruction and Development (IBRD) have their headquarters in Washington, but India's vote share in the IMF is not 10%.
Other Related Points
- Option 1 is incorrect because it suggests that both statements are correct, which is not true.
- Option 2 is incorrect because it suggests that both statements are incorrect, which is also not true.
- Option 4 is incorrect because it suggests that Statement (II) is correct, which is not true. The IMF focuses on promoting international monetary cooperation and exchange rate stability, while the IBRD focuses on providing loans and other financial assistance for development projects. India is a member of both organizations and has a significant role in shaping their policies and decisions.
Q71: Match List I with List II

Choose the correct answer from the options given below:
(a) (A) - (I), (B) - (II), (C) - (III), (D) - (IV)
(b) (A) - (IV), (B) - (III), (C) - (II), (D) - (I)
(c) (A) - (II), (B) - (I), (C) - (III), (D) - (IV)
(d) (A) - (IV). (B) - (I), (C) - (III), (D) - (II)
Ans: b
Sol: The correct answer is option 2, where (A) - (IV), (B) - (III), (C) - (II), (D) - (I).
- Mahad Satyagraha was a movement led by Dr. B.R. Ambedkar in 1927 to fight for the right of the Dalits to access water sources.
- Quit India Movement was a mass civil disobedience movement launched by Mahatma Gandhi in 1942 demanding an end to British rule in India.
- Non-Cooperation Movement was a movement launched by Gandhi in 1920 to boycott British goods and institutions.
- Surat split refers to the split in the Indian National Congress in 1907 between the moderates and the extremists.
Other Related Points
- Option 1 is incorrect as it matches Surat split with 1927, which is incorrect.
- Option 3 is incorrect as it matches Mahad Satyagraha with 1907, which is incorrect.
- Option 4 is incorrect as it matches Quit India Movement with 1919-21, which is incorrect.
Q72: Match List I with List II

Choose the correct answer from the options given below:
(a) (A) - (IV), (B) - (II), (C) - (I), (D) - (III)
(b) (A) - (III), (B) - (I), (C) - (II), (D) - (IV)
(c) (A) - (I), (B) - (IV), (C) - (III), (D) - (II)
(d) (A) - (IV), (B) - (III), (C) - (I), (D) - (II)
Ans: d
Sol: The correct answer is option 4, where (A) is matched with (IV), (B) with (III), (C) with (I), and (D) with (II).
Other Related Points
- Option 1 is incorrect as it matches (A) with (IV) which is correct, but (B) with (II) which is incorrect. Similarly, (C) is matched with (I) which is correct, but (D) is matched with (III) which is incorrect.
- Option 2 is incorrect as it matches (A) with (III) which is incorrect, and (B) with (I) which is correct. Similarly, (C) is matched with (II) which is correct, but (D) is matched with (IV) which is incorrect.
- Option 3 is incorrect as it matches (A) with (I) which is incorrect, and (B) with (IV) which is correct. Similarly, (C) is matched with (III) which is correct, but (D) is matched with (II) which is incorrect.
Therefore, option 4 is the correct answer as it matches all the books with their respective authors correctly.
Q73: Match List I with List II

Choose the correct answer from the options given below:
(a) (A) - (I), (B) - (II), (C) - (IV), (D) - (III)
(b) (A) - (I), (B) - (II), (C) - (III), (D) - (IV)
(c) (A) - (II), (B) - (I), (C) - (IV), (D) - (III)
(d) (A) - (II), (B) - (I), (C) - (III), (D) - (IV)
Ans: b
Sol: The correct answer is option 2, where (A) is matched with (I), (B) with (II), (C) with (III), and (D) with (IV). Other Related Points
- Option 1 is incorrect as it matches (A) with (I) and (D) with (III), which is incorrect.
- Option 3 is incorrect as it matches (A) with (II) and (C) with (IV), which is incorrect.
- Option 4 is incorrect as it matches (A) with (II) and (C) with (III), which is incorrect.
Overall, it is important to have a good understanding of the different schedules and provisions in the Indian Constitution to correctly match them.
Q74: Match List I with List II

Choose the correct answer from the options given below:
(a) (A) - (I), (B) - (II), (C) - (III), (D) - (IV)
(b) (A) - (IV), (B) - (I), (C) - (III), (D) - (II)
(c) (A) - (III). (B) - (I), (C) - (II), (D) - (IV)
(d) (A) - (II), (B) - (I), (C) - (IV), (D) - (III)
Ans: c
Sol: The correct answer is option 3, where (A) - (III), (B) - (I), (C) - (II), (D) - (IV).
- DMK was founded by C.N. Annadurai, AIDMK was founded by M.G. Ramachandran, TDP was founded by N.T. Ramarao, and PDP was founded by Mufti Mohammad Sayeed.
Other Related Points
- Option 1 is incorrect as it matches DMK with M.G. Ramachandran, which is incorrect.
- Option 2 is incorrect as it matches AIDMK with Mufti Mohammad Sayeed, which is incorrect.
- Option 4 is incorrect as it matches TDP with Mufti Mohammad Sayeed, which is incorrect. It is important to note the founder leaders of regional political parties as they play a significant role in shaping the ideology and direction of the party.
Q75: Match List I with List II

Choose the correct answer from the options given below:
(a) (A) - (I), (B) - (III), (C) - (II), (D) - (IV)
(b) (A) - (II), (B) - (III), (C) - (IV), (D) - (I)
(c) (A) - (II), (B) - (I), (C) - (IV), (D) - (III)
(d) (A) - (I), (B) - (III), (C) - (IV), (D) - (II)
Ans: c
Sol: The correct answer is: 'A) A-II, B-I, C-IV, D-III'.
A-II: Distinction between obedience and acceptance – Chester Barnard
- Chester Barnard, in his seminal work "The Functions of the Executive," highlighted the distinction between obedience and acceptance in an organizational context.
- According to Barnard, authority is effective only when subordinates accept it, which ties closely to the concept of the "zone of indifference."
- This concept underscores the behavioral aspect of management and organizational efficiency.
B-I: The distinction between economic man and administrative man – H. Simon
- Herbert Simon introduced the concept of the economic man versus the administrative man to illustrate decision-making in organizations.
- The economic man assumes perfect rationality and complete information, while the administrative man operates within bounded rationality, emphasizing realistic decision-making under constraints.
- This distinction forms the basis of Simon's critique of classical economic theories in management.
C-IV: The distinction between charismatic leader and legal-rational authority – Max Weber
- Max Weber, in his theory of authority, differentiated between traditional, charismatic, and legal-rational authority.
- Charismatic authority is based on personal traits and leadership, while legal-rational authority is derived from structured rules and norms.
- This classification remains foundational in understanding organizational and political structures.
D-III: Distinction between principles and mechanics of management – F.W. Taylor
- F.W. Taylor, the father of scientific management, emphasized the distinction between principles and mechanics in management to enhance efficiency.
- His focus on principles of management, such as time studies and standardization, distinguished scientific methods from traditional practices.
Other Related Points
Chester Barnard:
- His contributions to organizational theory emphasize the importance of informal organizations and cooperative systems.
- He believed that an organization's survival depends on its ability to achieve efficiency and effectiveness simultaneously.
H. Simon:
- Simon’s concept of bounded rationality challenged the classical notion of decision-making, emphasizing cognitive limitations and environmental influences.
- His work laid the foundation for behavioral decision-making models.
Max Weber:
- Weber’s contributions extended beyond authority, addressing bureaucracy's role in rational-legal societies and the challenges it poses.
- He emphasized systematic processes and predictability in administrative systems.
F.W. Taylor:
- Taylor’s scientific management principles revolutionized industrial processes, emphasizing efficiency and task standardization.
- His work faced criticism for its mechanistic view but remains influential in operations management.
Q76: Match List I with List II

Choose the correct answer from the options given below:
(a) (A) - (I), (B) - (II), (C) - (III), (D) - (IV)
(b) (A) - (II), (B) - (I), (C) - (IV), (D) - (III)
(c) (A) - (IV), (B) - (III), (C) - (II), (D) - (I)
(d) (A) - (III), (B) - (II), (C) - (I), (D) - (IV)
Ans: b
Sol: The correct answer is option 2, where (A) is matched with (II), (B) with (I), (C) with (IV), and (D) with (III).
Other Related Points
- Option 1 is incorrect as it matches (A) with (I) instead of (II), and (D) with (IV) instead of (III).
- Option 3 is incorrect as it matches (A) with (IV) instead of (II), and (B) with (III) instead of (I).
- Option 4 is incorrect as it matches (A) with (III) instead of (II), and (C) with (I) instead of (IV). It is important to carefully read and match the titles and authors in both lists to arrive at the correct answer.
Q77: Match List I with List II

Choose the correct answer from the options given below:
(a) (A) - (IV), (B) - (III), (C) - (I), (D) - (II)
(b) (A) - (II), (B) - (I), (C)-(III), (D) - (IV)
(c) (A) - (III), (B) - (II), (C)-(IV), (D) - (I)
(d) (A) - (I), (B) - (II), (C) - (III), (D) - (IV)
Ans: b
Sol: The correct answer is option 2: (A) - (II), (B) - (I), (C)-(III), (D) - (IV).
- This is because Plato believed that virtue is knowledge, while Machiavelli believed that success is virtue.
- Foucault believed that knowledge is power, and Russell believed that power is the production of intended effects.
Other Related Points
- Option 1 is incorrect because it switches the concepts of Machiavelli and Russell.
- Option 3 switches the concepts of Plato and Foucault.
- Option 4 correctly matches the concepts and thinkers.
Q78: Arrange the following Anti-Corruption measures taken by Government of India in chronological order
A. Ratification of the United Nations Convention Against Corruption (UNCAC)
B. Black Money (Undisclosed foreign income and assets) and Imposition of Tax Act
C. The Prevention of Money Laundering Act
D. The Benami Transactions (Prohibition) Act
Choose the correct answer from the options given below:
(a) (D), (A), (B), (C)
(b) (D), (C), (A), (B)
(c) (D), (C), (B), (A)
(d) (D), (B), (C), (A)
Ans: b
Sol: The correct answer is option 2, which is (D), (C), (A), (B).
- The Benami Transactions (Prohibition) Act was passed in 1988, followed by the Prevention of Money Laundering Act in 2002.
- India ratified the United Nations Convention Against Corruption (UNCAC) in 2011, and the Black Money (Undisclosed foreign income and assets) and Imposition of Tax Act was passed in 2015.
Other Related Points
- Option 1, (D), (A), (B), (C), is incorrect as the UNCAC was ratified after the Benami Transactions (Prohibition) Act.
- Option 3, (D), (C), (B), (A), is also incorrect as the Black Money (Undisclosed foreign income and assets) and Imposition of Tax Act was passed after the UNCAC was ratified.
- Option 4, (D), (B), (C), (A), is incorrect as the Prevention of Money Laundering Act was passed before the Black Money (Undisclosed foreign income and assets) and Imposition of Tax Act.
Q79: List chronologically the joint exercises conducted by Indian Air Force :
A. Indradhanush - IV
B. Desert Eagle - II
C. Table Top
D. Samvedna
Choose the correct answer from the options given below:
(a) (A), (B), (D) and (C)
(b) (B), (C), (D) and (A)
(c) (C), (D), (A) and (B)
(d) (D), (A), (B) and (C)
Ans: a
Sol: The correct answer is '(A), (B), (D) and (C)' which means that the correct order of the given styles is Indradhanush - IV, Desert Eagle - II, Samvedna and Table Top,
Indradhanush - IV
- The fourth edition of the exercise was held between 21–30 July 2015 at RAF Coningsby and RAF Brize Norton in United Kingdom. India was represented by four air superiority fighter Sukhoi Su-30 MKI's from No. 2 Squadron based at Tezpur AFS, one mid-air refueling tanker Ilyushin IL-78 MKI from No.
Desert Eagle - II
- The Exercise Desert Eagle is a bilateral exercise between India and the United Arab Emirates in 2016. India and the United Arab Emirates have had a strong bilateral relationship for hundreds of years.
Table Top
- A Table Top Exercise between the Indian Air Force (IAF) and the Myanmar Air Force (MAF) was conducted at Prayagraj on the theme of HADR in the year 2020.
Samvedna.
- The first Humanitarian Assistance and Disaster Relief (HADR) exercise Samvedna of the Indian Air Force in association with South Asian Region nations has started off the coast of Kerala from March 12,2018.
Q80: Arrange the following BRICS summit in a chronological order:
A. Ufa, Russia
B. Johannesberg, South Africa
C. Durban, South Africa
D. Beijing Virtual Summit
Choose the correct answer from the options given below:
(a) (C), (A), (B) and (D)
(b) (B), (C), (A) and (D)
(c) (A), (B), (C) and (D)
(d) (D), (B), (A) and (C)
Ans: a
Sol: The correct option is '(C), (A), (B) and (D)'.
Durban, South Africa -
- This summit was held in 2013. It was the fifth BRICS summit and took place in Durban, South Africa.
- The leaders discussed various topics including economic cooperation, global governance, and sustainable development.
Ufa, Russia -
- This summit was held in 2015. It marked the seventh BRICS summit and took place in Ufa, Russia.
- The leaders discussed regional and global issues, signed agreements on economic cooperation, and addressed terrorism and the situation in the Middle East.
Johannesburg, South Africa -
- This summit was held in 2018. It was the tenth BRICS summit and took place in Johannesburg, South Africa.
- The leaders focused on promoting inclusive growth, sustainable development, and advancing the Fourth Industrial Revolution.
- They also discussed global governance and international peace and security.
Beijing Virtual Summit -
- This virtual summit was held in 2022. It was organized due to the COVID-19 pandemic, and the leaders participated through video conferencing.
- The summit focused on joint efforts to combat the pandemic, enhance economic cooperation, and strengthen multilateralism.
Q81: Read the following passage and answer the questions that follow:
Authoritarianism is characterized by highly concentrated and centralized power maintained through political repression and exclusion of political challenges. It uses political parties and mass organization to mobilize people for the attainment of the goals of the regime. Authoritarianism demands unquestioning obedience to authority believing it is important for maintaining social order and preventing disruption. Several typologies have been developed to describe their morphological variations. Authoritarian regimes can be autocratic or oligarchic in nature, based on party, leader or military rule. Authoritarian governments often lack free and competitive, direct or indirect election of leaders' or both. They also check civil liberties such as freedom of speech and expression. Authoritarian state may include nominally democratic institutions such as political parties, parliaments and elections designed to legitimize dictatorship through deceptive and uncompetitive elections.
Which of the following is not a characteristic of an authoritarian regime?
(a) Nominally democratic institutions.
(b) Uncompetitive election of leadership.
(c) Mobilisation of people for attainment of regime's goals.
(d) Use of mass organization to strengthen democracy.
Ans: d
Sol: Option 4 is the correct answer because the use of mass organization to strengthen democracy is not a characteristic of an authoritarian regime.
- Authoritarian regimes typically do not promote democracy or allow for the free organization of groups that could potentially challenge their power.
Other Related Points
- Option 1 refers to the presence of democratic institutions, which authoritarian regimes may have in name only, but in practice, these institutions are often controlled by the ruling party or leader.
- Option 2 refers to the lack of competition in the election of leadership, which is a common feature of authoritarian regimes where the ruling party or leader has a monopoly on power.
- Option 3 refers to the mobilization of people to support the regime's goals, which is often achieved through propaganda, coercion, or manipulation of the media. This is a common tactic used by authoritarian regimes to maintain their grip on power.
Q82: Read the following passage and answer the questions that follow:
Authoritarianism is characterized by highly concentrated and centralized power maintained through political repression and exclusion of political challenges. It uses political parties and mass organization to mobilize people for the attainment of the goals of the regime. Authoritarianism demands unquestioning obedience to authority believing it is important for maintaining social order and preventing disruption. Several typologies have been developed to describe their morphological variations. Authoritarian regimes can be autocratic or oligarchic in nature, based on party, leader or military rule. Authoritarian governments often lack free and competitive, direct or indirect election of leaders' or both. They also check civil liberties such as freedom of speech and expression. Authoritarian state may include nominally democratic institutions such as political parties, parliaments and elections designed to legitimize dictatorship through deceptive and uncompetitive elections.
Given below are two statements :
Statement (I): Authoritarianism demands unquestioning obedience to authority.
Statement (II): Authoritarian governments often lack free and competitive election of leaders.
In the light of the above statements, choose the correct answer from the options given below:
(a) Both Statement (I) and Statement (II) are true
(b) Both Statement (I) and Statement (II) are false
(c) Statement (I) is true but Statement (II) is false
(d) Statement (I) is false but Statement (II) is true
Ans: a
Sol: The correct answer is option 1 - Both Statement (I) and Statement (II) are true. Statement (I) is true because authoritarianism is characterized by a concentration of power in the hands of a single leader or ruling group, who demand obedience and loyalty from their subjects.
- Dissent and opposition are often suppressed, and individual freedoms and rights are curtailed. Statement (II) is also true because authoritarian governments often lack free and competitive elections, which are a hallmark of democratic societies.
- Instead, leaders are often appointed or selected through non-democratic means, such as inheritance, military coup, or rigged elections.
Other Related Points
- Option 2 - Both Statement (I) and Statement (II) are false - is incorrect because both statements are true.
- Option 3 - Statement (I) is true but Statement (II) is false - is incorrect because Statement (II) is also true.
- Option 4 - Statement (I) is false but Statement (II) is true - is incorrect because Statement (I) is true.
Overall, the correct answer is option 1, as both statements accurately describe the characteristics of authoritarianism and authoritarian governments.
Q83: Read the following passage and answer the questions that follow:
Authoritarianism is characterized by highly concentrated and centralized power maintained through political repression and exclusion of political challenges. It uses political parties and mass organization to mobilize people for the attainment of the goals of the regime. Authoritarianism demands unquestioning obedience to authority believing it is important for maintaining social order and preventing disruption. Several typologies have been developed to describe their morphological variations. Authoritarian regimes can be autocratic or oligarchic in nature, based on party, leader or military rule. Authoritarian governments often lack free and competitive, direct or indirect election of leaders' or both. They also check civil liberties such as freedom of speech and expression. Authoritarian state may include nominally democratic institutions such as political parties, parliaments and elections designed to legitimize dictatorship through deceptive and uncompetitive elections.
Why authoritarian regimes command unreasoned acceptance of their political authority?
(a) To strengthen democratic ideals
(b) For enhancing civil liberties
(c) For constitutional governance
(d) For preventing political disruption
Ans: d
Sol: The correct answer is option 4 - for preventing political disruption.
- Authoritarian regimes often rely on fear and intimidation to maintain their power, and unreasoned acceptance of their political authority helps to prevent any challenges or disruptions to their rule.
- By discouraging dissent and suppressing opposition, authoritarian leaders can maintain control over their population and prevent any potential threats to their regime.
Other Related Points
- Option 1 - strengthening democratic ideals - is not applicable to authoritarian regimes, as they do not prioritize democratic values or principles.
- Option 2 - enhancing civil liberties - is also not applicable, as authoritarian regimes often restrict or violate civil liberties in order to maintain their power.
- Option 3 - constitutional governance - may be a stated goal of some authoritarian regimes, but in practice, they often disregard constitutional norms and principles in favor of consolidating their own power.
Q84: Read the following passage and answer the questions that follow:
Authoritarianism is characterized by highly concentrated and centralized power maintained through political repression and exclusion of political challenges. It uses political parties and mass organization to mobilize people for the attainment of the goals of the regime. Authoritarianism demands unquestioning obedience to authority believing it is important for maintaining social order and preventing disruption. Several typologies have been developed to describe their morphological variations. Authoritarian regimes can be autocratic or oligarchic in nature, based on party, leader or military rule. Authoritarian governments often lack free and competitive, direct or indirect election of leaders' or both. They also check civil liberties such as freedom of speech and expression. Authoritarian state may include nominally democratic institutions such as political parties, parliaments and elections designed to legitimize dictatorship through deceptive and uncompetitive elections.
Authoritarian regimes legitimise their dictatorship through which of the below methods?
A. Through periodic fair elections.
B. Through competitive election of leaders.
C. Through deceptive and indirect elections.
D. Through blocking opportunities to potential challenges
Choose the most appropriate answer from the options given below:
(a) (A) and (B) only
(b) (B) and (C) only
(c) (C) and (D) only
(d) (A) and (D) only
Ans: c
Sol: The correct answer is option 3, which states that authoritarian regimes legitimise their dictatorship through deceptive and indirect elections and blocking opportunities to potential challenges.
- This means that these regimes may hold elections, but they are often rigged or manipulated to ensure the ruling party or leader remains in power.
- They may also use tactics such as censorship, intimidation, and violence to prevent opposition parties or individuals from gaining power or challenging their authority.
Other Related Points
- Option 1, which states that authoritarian regimes legitimise their dictatorship through periodic fair elections, is incorrect because these regimes often do not hold truly fair elections.
- Option 2, which states that they legitimise their dictatorship through competitive election of leaders, is also incorrect because these regimes often do not allow for true competition or may manipulate the election process to ensure their preferred candidate wins.
- Option 4, which states that they legitimise their dictatorship through blocking opportunities to potential challenges, is partially correct but does not include the role of deceptive and indirect elections in legitimising their rule.
Q85: Read the following passage and answer the questions that follow:
Authoritarianism is characterized by highly concentrated and centralized power maintained through political repression and exclusion of political challenges. It uses political parties and mass organization to mobilize people for the attainment of the goals of the regime. Authoritarianism demands unquestioning obedience to authority believing it is important for maintaining social order and preventing disruption. Several typologies have been developed to describe their morphological variations. Authoritarian regimes can be autocratic or oligarchic in nature, based on party, leader or military rule. Authoritarian governments often lack free and competitive, direct or indirect election of leaders' or both. They also check civil liberties such as freedom of speech and expression. Authoritarian state may include nominally democratic institutions such as political parties, parliaments and elections designed to legitimize dictatorship through deceptive and uncompetitive elections.
What remains the status of civil liberties in authoritarian regimes?
(a) Remain as freely available as in democratic regimes.
(b) Their status remains unchanged.
(c) They are respected more.
(d) All the above are false.
Ans: d
Sol: The correct answer is option 4, which states that all the above options are false.
- In authoritarian regimes, civil liberties are often severely restricted or completely absent.
- The government has complete control over the media, freedom of speech, assembly, and association are limited, and political opposition is often suppressed.
- Citizens may also face arbitrary detention, torture, and other forms of human rights abuses.
Therefore, options 1, 2, and 3 are all incorrect as civil liberties are not freely available, their status is not unchanged, and they are not respected more in authoritarian regimes.
Q86: Read the following passage and answer the questions that follow:
Public administration is a discipline in transition. In fact, it has always been in continuous movement, but not always in the same direction. Contrary to the heavy, formal, and inflexible image of bureaucracies, public sector bodies in America, Europe, and elsewhere have been in a rapidly intensifying transition since the early 1990s. During the last century, public administration underwent significant changes resulting from crisis, as well as breakthroughs in an ultra dynamic environment. Some 30 years ago, Waldo (1968) noted that these ongoing transformations reflected an identity crisis of a science in formation. They also signaled a struggle for the recognition and legitimacy of public administration as an art, a body of knowledge and a profession (Lynn, 1996). It seems that today, at the beginning of the 21st century, the formation of public administration is still unfinished business. it is a subject for debate among academics and practitioners across the world who seek higher and more extensive scientific recognition, more accurate self-definition, and better applicability of the field to rapid changes in modern life. This process presents new challenges for public administration. Perhaps, the most important is to integrate more widely existing knowledge of the social sciences with efficient public action and with quality governmental cooperation. In the coming years public administration will be evaluated by higher stands of theory cohesiveness and by more comprehensive performance indicators rooted in a variety of scientific fields.
To talk of the 'Public' of 'Administration', and of the integration of the two constructs into a useful terrain for study involves promises as well as difficulties. But consensus does exist on at least one issue: the public needs a better bureaucracy. The public also expects good and skillful administrators, familiar with the mysteries of quality services and effective management. Only they can produce better 'public goods' and deliver them to all sectors of society with minimum time and cost. This is a revised version of the ideal type of public administration system applicable to modern times.
The role of the state and its relationship to bureaucracy and to citizens are undergoing substantial transformation, not only in the minds of the people but also in scientific thinking. In a rapidly changing environment, public administration has a major function and new aims that must be clearly recognized.
Identify the correct option among the following statements related to the discipline of Public Administration.
A. Public expects good and skillful administrator
B. Public expects quality of services
C. Public expects effective management of institution
D. Public wants to continue with the present model of bureaucracy
Choose the correct answer from the options given below:
(a) (A), (B) and (C) only
(b) (A), (B) and (D) only
(c) (A), (C) and (D) only
(d) (B), (C) and (D) only
Ans: a
Sol: The correct answer is option 1, which includes statements (A), (B), and (C) only.
- This is because the discipline of Public Administration is focused on meeting the needs and expectations of the public, which includes providing good and skillful administrators, quality services, and effective management of institutions.
Other Related Points
- Option 2 is incorrect because the public does not necessarily want to continue with the present model of bureaucracy.
- Option 3 is incorrect because statement (D) is not aligned with the expectations of the public.
- Option 4 is incorrect because statement (A) is also an important expectation of the public.
Q87: Read the following passage and answer the questions that follow:
Public administration is a discipline in transition. In fact, it has always been in continuous movement, but not always in the same direction. Contrary to the heavy, formal, and inflexible image of bureaucracies, public sector bodies in America, Europe, and elsewhere have been in a rapidly intensifying transition since the early 1990s. During the last century, public administration underwent significant changes resulting from crisis, as well as breakthroughs in an ultra dynamic environment. Some 30 years ago, Waldo (1968) noted that these ongoing transformations reflected an identity crisis of a science in formation. They also signaled a struggle for the recognition and legitimacy of public administration as an art, a body of knowledge and a profession (Lynn, 1996). It seems that today, at the beginning of the 21st century, the formation of public administration is still unfinished business. it is a subject for debate among academics and practitioners across the world who seek higher and more extensive scientific recognition, more accurate self-definition, and better applicability of the field to rapid changes in modern life. This process presents new challenges for public administration. Perhaps, the most important is to integrate more widely existing knowledge of the social sciences with efficient public action and with quality governmental cooperation. In the coming years public administration will be evaluated by higher stands of theory cohesiveness and by more comprehensive performance indicators rooted in a variety of scientific fields.
To talk of the 'Public' of 'Administration', and of the integration of the two constructs into a useful terrain for study involves promises as well as difficulties. But consensus does exist on at least one issue: the public needs a better bureaucracy. The public also expects good and skillful administrators, familiar with the mysteries of quality services and effective management. Only they can produce better 'public goods' and deliver them to all sectors of society with minimum time and cost. This is a revised version of the ideal type of public administration system applicable to modern times.
The role of the state and its relationship to bureaucracy and to citizens are undergoing substantial transformation, not only in the minds of the people but also in scientific thinking. In a rapidly changing environment, public administration has a major function and new aims that must be clearly recognized.
What is the main reason for a major role of Public Administration in future?
(a) Scientific thinking among people
(b) Citizens are static
(c) Citizens are more active due to political complexity
(d) Fall of relation between state and citizen
Ans: a
Sol: The correct answer is option 1, scientific thinking among people.
- This is because as society becomes more complex and interconnected, there is a greater need for evidence-based decision making and problem solving
- Public administration plays a crucial role in this process by providing the necessary expertise and resources to analyze and address complex issues facing society.
Other Related Points
- Option 2, citizens are static, is incorrect because citizens are not static and their needs and expectations are constantly evolving. Public administration must adapt to these changes in order to effectively serve the public.
- Option 3, citizens are more active due to political complexity, is partially correct. As political complexity increases, citizens may become more engaged and demand greater accountability from their government. However, this alone is not the main reason for the major role of public administration in the future.
- Option 4, fall of relation between state and citizen, is incorrect. While there may be challenges in the relationship between the state and citizens, public administration plays a critical role in bridging this gap and ensuring that the needs and interests of citizens are represented in government decision making.
Overall, the main reason for the major role of public administration in the future is the need for scientific thinking and evidence-based decision making in an increasingly complex and interconnected society.
Q88: Read the following passage and answer the questions that follow:
Public administration is a discipline in transition. In fact, it has always been in continuous movement, but not always in the same direction. Contrary to the heavy, formal, and inflexible image of bureaucracies, public sector bodies in America, Europe, and elsewhere have been in a rapidly intensifying transition since the early 1990s. During the last century, public administration underwent significant changes resulting from crisis, as well as breakthroughs in an ultra dynamic environment. Some 30 years ago, Waldo (1968) noted that these ongoing transformations reflected an identity crisis of a science in formation. They also signaled a struggle for the recognition and legitimacy of public administration as an art, a body of knowledge and a profession (Lynn, 1996). It seems that today, at the beginning of the 21st century, the formation of public administration is still unfinished business. it is a subject for debate among academics and practitioners across the world who seek higher and more extensive scientific recognition, more accurate self-definition, and better applicability of the field to rapid changes in modern life. This process presents new challenges for public administration. Perhaps, the most important is to integrate more widely existing knowledge of the social sciences with efficient public action and with quality governmental cooperation. In the coming years public administration will be evaluated by higher stands of theory cohesiveness and by more comprehensive performance indicators rooted in a variety of scientific fields.
To talk of the 'Public' of 'Administration', and of the integration of the two constructs into a useful terrain for study involves promises as well as difficulties. But consensus does exist on at least one issue: the public needs a better bureaucracy. The public also expects good and skillful administrators, familiar with the mysteries of quality services and effective management. Only they can produce better 'public goods' and deliver them to all sectors of society with minimum time and cost. This is a revised version of the ideal type of public administration system applicable to modern times.
The role of the state and its relationship to bureaucracy and to citizens are undergoing substantial transformation, not only in the minds of the people but also in scientific thinking. In a rapidly changing environment, public administration has a major function and new aims that must be clearly recognized.
Who among the following has identified the transition of Public Administration to a profession?
(a) Dwight Waldo
(b) Woodrow Wilson
(c) Lynn
(d) F.M. Marx
Ans: c
Sol: The correct answer is option 3, Lynn.
- Lynn was a scholar who identified the transition of Public Administration to a profession. He argued that Public Administration had evolved from a political activity to a professional one, with its own set of principles, values, and techniques.
- He also emphasized the importance of education and training for public administrators.
Other Related Points
- Option 1, Dwight Waldo, was also a prominent scholar in the field of Public Administration. He is known for his work on the study of administrative ethics and the role of the public administrator in society.
- Option 2, Woodrow Wilson, is often credited with laying the foundation for the study of Public Administration. In his famous essay, "The Study of Administration," he argued for the professionalization of public service and the need for trained experts to manage government.
- Option 4, F.M. Marx, is not a well-known figure in the field of Public Administration. It is unclear what his contributions to the field were.
Overall, the study of Public Administration has evolved over time, with various scholars contributing to its development and professionalization. Lynn's identification of the transition to a profession is just one example of this ongoing evolution.
Q89: Read the following passage and answer the questions that follow:
Public administration is a discipline in transition. In fact, it has always been in continuous movement, but not always in the same direction. Contrary to the heavy, formal, and inflexible image of bureaucracies, public sector bodies in America, Europe, and elsewhere have been in a rapidly intensifying transition since the early 1990s. During the last century, public administration underwent significant changes resulting from crisis, as well as breakthroughs in an ultra dynamic environment. Some 30 years ago, Waldo (1968) noted that these ongoing transformations reflected an identity crisis of a science in formation. They also signaled a struggle for the recognition and legitimacy of public administration as an art, a body of knowledge and a profession (Lynn, 1996). It seems that today, at the beginning of the 21st century, the formation of public administration is still unfinished business. it is a subject for debate among academics and practitioners across the world who seek higher and more extensive scientific recognition, more accurate self-definition, and better applicability of the field to rapid changes in modern life. This process presents new challenges for public administration. Perhaps, the most important is to integrate more widely existing knowledge of the social sciences with efficient public action and with quality governmental cooperation. In the coming years public administration will be evaluated by higher stands of theory cohesiveness and by more comprehensive performance indicators rooted in a variety of scientific fields.
To talk of the 'Public' of 'Administration', and of the integration of the two constructs into a useful terrain for study involves promises as well as difficulties. But consensus does exist on at least one issue: the public needs a better bureaucracy. The public also expects good and skillful administrators, familiar with the mysteries of quality services and effective management. Only they can produce better 'public goods' and deliver them to all sectors of society with minimum time and cost. This is a revised version of the ideal type of public administration system applicable to modern times.
The role of the state and its relationship to bureaucracy and to citizens are undergoing substantial transformation, not only in the minds of the people but also in scientific thinking. In a rapidly changing environment, public administration has a major function and new aims that must be clearly recognized.
Which among the following statements are not related to the transition of the discipline of Public Administration?
(a) Accurate self-definition
(b) Changes in the modern life
(c) Extensive scientific recognition
(d) Economic and Financial Crisis of the Markets
Ans: d
Sol: The correct answer is Economic and Financial Crisis of the Markets, is not related to the transition of the discipline of Public Administration.
- This statement is more related to the field of economics and finance.
Other Related Points
- Option 1, Accurate self-definition, refers to the process of defining the scope and boundaries of the discipline of Public Administration. This has been an ongoing process as the field has evolved over time.
- Option 2, Changes in the modern life, refers to the impact of technological advancements, globalization, and other societal changes on the practice of Public Administration.
- Option 3, Extensive scientific recognition, refers to the increasing use of empirical research and evidence-based practices in the field of Public Administration. This has led to a greater emphasis on data-driven decision making and evaluation.
Q90: Read the following passage and answer the questions that follow:
Public administration is a discipline in transition. In fact, it has always been in continuous movement, but not always in the same direction. Contrary to the heavy, formal, and inflexible image of bureaucracies, public sector bodies in America, Europe, and elsewhere have been in a rapidly intensifying transition since the early 1990s. During the last century, public administration underwent significant changes resulting from crisis, as well as breakthroughs in an ultra dynamic environment. Some 30 years ago, Waldo (1968) noted that these ongoing transformations reflected an identity crisis of a science in formation. They also signaled a struggle for the recognition and legitimacy of public administration as an art, a body of knowledge and a profession (Lynn, 1996). It seems that today, at the beginning of the 21st century, the formation of public administration is still unfinished business. it is a subject for debate among academics and practitioners across the world who seek higher and more extensive scientific recognition, more accurate self-definition, and better applicability of the field to rapid changes in modern life. This process presents new challenges for public administration. Perhaps, the most important is to integrate more widely existing knowledge of the social sciences with efficient public action and with quality governmental cooperation. In the coming years public administration will be evaluated by higher stands of theory cohesiveness and by more comprehensive performance indicators rooted in a variety of scientific fields.
To talk of the 'Public' of 'Administration', and of the integration of the two constructs into a useful terrain for study involves promises as well as difficulties. But consensus does exist on at least one issue: the public needs a better bureaucracy. The public also expects good and skillful administrators, familiar with the mysteries of quality services and effective management. Only they can produce better 'public goods' and deliver them to all sectors of society with minimum time and cost. This is a revised version of the ideal type of public administration system applicable to modern times.
The role of the state and its relationship to bureaucracy and to citizens are undergoing substantial transformation, not only in the minds of the people but also in scientific thinking. In a rapidly changing environment, public administration has a major function and new aims that must be clearly recognized.
In the light of the above statements, choose the most appropriate answer from the options given below:
(a) Both (A) and (R) are correct and (R) is the correct explanation of (A)
(b) Both (A) and (R) are correct but (R) is NOT the correct explanation of (A)
(c) (A) is correct but (R) is not correct
(d) (A) is not correct but (R) is correct
Ans: a
Sol: The correct answer is option 1 - Both (A) and (R) are correct and (R) is the correct explanation of (A).
- The assertion states that in the future, Public Administration will be assessed with scientific tools such as performance indicators and high standards of theory.
- This is because the impact of public choice and New Public Management (NPM) is more on present-day administration.
- The reason given explains why the assertion is true. Public choice theory emphasizes the importance of using economic principles to analyze public decision-making, while NPM emphasizes the need for efficiency and effectiveness in public service delivery. Both of these approaches require the use of scientific tools to measure performance and outcomes.
Other Related Points
- Option 2 - Both (A) and (R) are correct but (R) is NOT the correct explanation of (A) - is incorrect because the reason given does explain why the assertion is true.
- Option 3 - (A) is correct but (R) is not correct - is incorrect because the reason given is correct.
- Option 4 - (A) is not correct but (R) is correct - is incorrect because the assertion is correct. Option 5 - The correct answer is option 1.
Q91: Arrange the following concepts in chronological order according to their evolution.
A. Sampurna Kranti
B. The saptang theory of State
C. Radical Humanism
D. Satyagrah
Choose the correct answer from the options given below :
(a) (B), (C), (D), (A)
(b) (A), (D), (C), (B)
(c) (D), (A), (B), (C)
(d) (A), (C), (D), (B)
Ans: a
Sol: The correct answer is option 1, which is (B), (C), (D), (A).
- The saptang theory of State was developed in ancient India and was followed by the concept of Radical Humanism, which emerged during the Indian independence movement.
- Satyagraha, which means non-violent resistance, was a key strategy used during the independence movement.
- Finally, Sampurna Kranti, which means total revolution, was a concept that emerged after India gained independence and aimed to bring about social and economic change in the country.
Other Related Points
- Option 2, which is (A), (D), (C), (B), is incorrect because it places Satyagraha before Radical Humanism, which is historically inaccurate.
- Option 3, which is (D), (A), (B), (C), is incorrect because it places Satyagraha before the saptang theory of State, which is also historically inaccurate.
- Option 4, which is (A), (C), (D), (B), is incorrect because it places Radical Humanism before Satyagraha, which is not accurate as Satyagraha was a key strategy used during the independence movement and preceded the concept of Radical Humanism.
Q92: Arrange the following movements in chronological order.
A. Chipko Movement
B. Jal Satyagrah
C. Appiko Movement
D. Jan Lokpal Movement
E. Narmada Bachao Andolan
Choose the correct answer from the options given below:
(a) (B), (C), (D), (E), (A)
(b) (A), (C), (E), (D), (B)
(c) (D), (C), (A), (E), (B)
(d) (C), (A), (D), (B), (E)
Ans: b
Sol: The correct answer is (A), (C), (E), (D), (B).
The Chipko Movement started in 1973, the Appiko Movement started in 1983, the Narmada Bachao Andolan started in 1985, the Jan Lokpal Movement started in 2011, and the Jal Satyagraha started in 2012.
- The Chipko Movement was a nonviolent social and ecological movement that started in 1973 in the Garhwal region of Uttarakhand, then a part of Uttar Pradesh (at the foothills of the Himalayas) in India.
- The movement was against the indiscriminate felling of trees and the destruction of forests.
- The movement was started by women who hugged the trees to prevent them from being cut down.
- The Appiko Movement was a nonviolent social and ecological movement that started in 1983 in the Uttara Kannada district of Karnataka, India.
- The movement was against the indiscriminate felling of trees and the destruction of forests.
- The movement was started by men and women who hugged the trees to prevent them from being cut down.
- The Narmada Bachao Andolan is a nonviolent social and ecological movement that started in 1985 in the Narmada valley of Madhya Pradesh, India.
- The movement is against the construction of large dams on the Narmada river.
- The movement has been successful in stopping the construction of some dams and delaying the construction of others.
- The Jan Lokpal Movement was a nonviolent social and political movement that started in 2011 in India.
- The movement was against corruption in the government.
- The movement was successful in passing the Lokpal and Lokayuktas Act, 2013, which established the Lokpal, an anti-corruption ombudsman.
- The Jal Satyagraha is a nonviolent social and political movement that started in 2012 in India.
- The movement is against the displacement of people due to the construction of large dams.
- The movement has been successful in stopping the construction of some dams and delaying the construction of others.
Q93: Arrange the following experiments of Elton Mayo in chronological order.
A. Relay Assembly Test Room Experiment
B. Illumination Experiment
C. Mass Interview Programme
D. Bank Wiring Experiment
Choose the correct answer from the options given below:
(a) (C), (A), (B) and (D)
(b) (B), (A), (C) and (D)
(c) (C), (B), (A) and (D)
(d) (D), (A), (C) and (B)
Ans: b
Sol: The correct answer is option 2, which arranges the experiments in the following chronological order: (B) Illumination Experiment, (A) Relay Assembly Test Room Experiment, (C) Mass Interview Programme, and (D) Bank Wiring Experiment.
- The Illumination Experiment was conducted in 1924 and was the first experiment conducted by Mayo and his team.
- The Relay Assembly Test Room Experiment was conducted in 1927-1932 and was the most famous experiment conducted by Mayo.
- The Mass Interview Programme was conducted in 1928-1930 and involved interviewing workers to understand their attitudes towards work.
- The Bank Wiring Experiment was conducted in 1931 and was the last experiment conducted by Mayo and his team.
Other Related Points
- Option 1 arranges the experiments in the order of the Mass Interview Programme, Relay Assembly Test Room Experiment, Illumination Experiment, and Bank Wiring Experiment.
- Option 3 arranges the experiments in the order of the Mass Interview Programme, Illumination Experiment, Relay Assembly Test Room Experiment, and Bank Wiring Experiment.
- Option 4 arranges the experiments in the order of the Bank Wiring Experiment, Relay Assembly Test Room Experiment, Mass Interview Programme, and Illumination Experiment.
Q94: Match List I with List II

Choose the correct answer from the options given below:
(a) (A) - (II), (B) - (III), (C) - (IV), (D) - (I)
(b) (A) - (III), (B) - (I), (C) - (II), (D) - (IV)
(c) (A) - (I), (B) - (II), (C) - (III), (D) - (IV)
(d) (A) - (IV), (B) - (II), (C) - (I), (D) - (III)
Ans: c
Sol: The correct answer is option 3, which matches the years and summits correctly.
The Bandung Summit took place in 1955, the Belgrade Summit in 1961, the New Delhi Summit in 1983, and the Durban Summit in 1998.
- Option 1 is incorrect as it matches the years and summits incorrectly.
- Option 2 is also incorrect as it matches the years and summits incorrectly.
- Option 4 is incorrect as it matches the years and summits incorrectly.
Other Related Points
- The Bandung Summit was a meeting of Asian and African states in Indonesia, which aimed to promote economic and cultural cooperation and oppose colonialism and imperialism.
- The Belgrade Summit was a meeting of non-aligned countries, which aimed to promote cooperation and independence from the superpowers during the Cold War.
- The New Delhi Summit was a meeting of non-aligned countries, which aimed to address issues such as disarmament, economic development, and human rights.
- The Durban Summit was a meeting of the United Nations World Conference Against Racism, which aimed to address issues of racism, discrimination, and xenophobia.
Q95: Arrange the concepts in chronological order according to evolution.
A. Legal Sovereignty
B. Popular Sovereignty
C. Political Sovereignty
D. Theory of Plural Sovereignty
Choose the correct answer from the options given below:
(a) (A), (D), (C), (B)
(b) (A), (B), (D), (C)
(c) (A), (C), (B), (D)
(d) (D), (B), (A), (C)
Ans: c
Sol: The correct answer is option 3, which arranges the concepts in chronological order according to evolution as follows:
- Legal Sovereignty, Political Sovereignty, Popular Sovereignty, and Theory of Plural Sovereignty.
Other Related Points
- Option 1 arranges the concepts in the order of Legal Sovereignty, Theory of Plural Sovereignty, Political Sovereignty, and Popular Sovereignty, which is incorrect.
- Option 2 arranges the concepts in the order of Legal Sovereignty, Popular Sovereignty, Theory of Plural Sovereignty, and Political Sovereignty, which is also incorrect.
- Option 4 arranges the concepts in the order of Theory of Plural Sovereignty, Popular Sovereignty, Legal Sovereignty, and Political Sovereignty, which is again incorrect.
Q96: Arrange the following books according to their publication year in chronological order
A. The Archaeology of Knowledge
B. The Human Condition
C. Spheres of Justice
D. After Virtue
Choose the correct answer from the options given below:
(a) (B), (A), (D), (C)
(b) (A), (B), (C), (D)
(c) (D), (A), (B), (C)
(d) (C), (D), (A), (B)
Ans: a
Sol: The correct answer is option 1: (B), (A), (D), (C).
- The Human Condition by Hannah Arendt was published in 1958, making it the oldest book on the list.
- The Archaeology of Knowledge by Michel Foucault was published in 1969, followed by After Virtue by Alasdair MacIntyre in 1981, and finally Spheres of Justice by Michael Walzer in 1983.
Other Related Points
- Option 2: (A), (B), (C), (D) is incorrect because it places The Archaeology of Knowledge before The Human Condition, which is incorrect chronologically.
- Option 3: (D), (A), (B), (C) is incorrect because it places After Virtue before The Archaeology of Knowledge, which is incorrect chronologically.
- Option 4: (C), (D), (A), (B) is incorrect because it places Spheres of Justice before The Archaeology of Knowledge, which is incorrect chronologically.
Q97: Match List I with List II

Choose the correct answer from the options given below:
(a) (A) - (II), (B) - (III), (C) - (I), (D) - (IV)
(b) (A) - (I), (B) - (III), (C) - (IV), (D) - (II)
(c) (A) - (III), (B) - (I), (C) - (IV), (D) - (II)
(d) (A) - (III), (B) - (I), (C) - (II), (D) - (IV)
Ans: d
Sol: The correct answer is option 4, where (A) - (III), (B) - (I), (C) - (II), (D) - (IV).
- UJJAWALA is a scheme under the Ministry of Petroleum and Natural Gas, which aims to provide free LPG connections to women from below poverty line households.
- Saubhagya Yojana is a scheme under the Ministry of Power, which aims to provide electricity connections to all households in rural and urban areas.
- Atal Pension Yojana is a scheme under the Ministry of Finance, which aims to provide a pension to workers in the unorganized sector.
- National Social Assistance Programme is a scheme under the Ministry of Rural Development, which aims to provide financial assistance to the elderly, widows, and disabled individuals living below the poverty line.
Other Related Points
- Option 1 is incorrect as it incorrectly matches UJJAWALA with the Ministry of Power.
- Option 2 is incorrect as it incorrectly matches Saubhagya Yojana with the Ministry of Finance.
- Option 3 is incorrect as it incorrectly matches UJJAWALA with the Ministry of Women and Child Development.
Q98: Match List I with List II

Choose the correct answer from the options given below:
(a) (A) - (II), (B) - (I), (C) - (III), (D) - (IV)
(b) (A) - (I), (B) - (II), (C) - (IV), (D) - (III)
(c) (A) - (IV), (B) - (III), (C) - (II), (D) - (I)
(d) (A) - (III), (B) - (IV), (C) - (I), (D) - (II)
Ans: a
Sol: The correct answer is option 1, where (A) - (II), (B) - (I), (C) - (III), (D) - (IV).
- Indian Peace Keeping Force in Sri Lanka was sent by Rajiv Gandhi, as part of the Indo-Sri Lanka Accord signed in 1987.
- The Shimla Agreement, brokered by Indira Gandhi, sought to end hostilities and normalize relations between India and Pakistan after the 1971 war but failed to resolve the Kashmir issue, perpetuating tensions.
- The Lahore Declaration was signed by Atal B. Vajpayee during his tenure as Prime Minister.
- Act East Policy is a foreign policy initiative of Narendra Modi, aimed at strengthening India's ties with Southeast Asian countries.
Other Related Points
- Option 2 is incorrect as it has interchanged the positions of (A) and (B).
- Option 3 is incorrect as it has interchanged the positions of (A) and (D).
- Option 4 is incorrect as it has interchanged the positions of (A) and (C).
Q99: Chronologically arrange the features of constitutionalism as they developed over period of time.
A. Checks and Balances
B. Codification of Law
C. Political Separatism
D. Constitutional Monarchy
Choose the correct answer from the options given below:
(a) (C), (B), (A), (D)
(b) (A), (B), (C), (D)
(c) (B), (C), (A), (D)
(d) (C), (B), (D), (A)
Ans: D
Sol: The correct answer is option 4, which is (C), (B), (D), (A).
- This is because political separatism, or the idea of separating powers between different branches of government, was the first feature to develop.
- This was followed by the codification of law, which involved creating written constitutions and laws that were binding on all citizens.
- The third feature to develop was constitutional monarchy, which limited the power of the monarch and gave more power to the people.
- Finally, checks and balances were added to ensure that no one branch of government had too much power.
Other Related Points
- Option 1, (C), (B), (A), (D), is incorrect because it places checks and balances before constitutional monarchy, which is historically inaccurate.
- Option 2, (A), (B), (C), (D), is incorrect because it places checks and balances first, which is not historically accurate.
- Option 3, (B), (C), (A), (D), is incorrect because it places the codification of law before political separatism, which is not historically accurate.
Q100: Arrange the following directive principles of state policy in a sequential order as mentioned in the Constitution of India.
A. Equal pay for equal work for men and women
B. To secure a uniform civil code throughout the country for all citizen
C. To promote equal justice and to provide free legal aid to the poor
D. Separate the judiciary from the executive in the public services
Choose the correct answer from the options given below:
(a) (A), (B), (C) and (D)
(b) (A), (C), (B) and (D)
(c) (C), (A), (B) and (D)
(d) (D), (B), (C) and (A)
Ans: b
Sol: The correct answer is (A), (C), (B) and (D)
A (Article 39(d)), C (Article 39A), B (Article 44), D (Article 50).
- Equal pay for equal work for men and women:
- This directive principle stresses the principle of gender equality in the workplace.
- It calls for the elimination of gender-based wage discrimination, ensuring that both men and women receive equal pay for doing the same work.
- To promote equal justice and to provide free legal aid to the poor:
- This directive principle emphasizes the promotion of equal justice and the provision of free legal aid to those who cannot afford it.
- It reflects the commitment of the state to ensure that justice is accessible to all, regardless of their socio-economic background.
- To secure a uniform civil code throughout the country for all citizens:
- This directive principle advocates for the implementation of a uniform civil code that applies to all citizens of India.
- A uniform civil code would provide a common set of laws governing personal matters such as marriage, divorce, inheritance, and adoption for people of all religions and communities.
- Separate the judiciary from the executive in the public services:
- This directive principle highlights the importance of maintaining a separation between the judiciary and the executive branches of government.
- It emphasizes the need for an independent judiciary that is free from political interference.